AP
®
United States
History
Scoring Guidelines
2019
© 2019 The College Board. College Board, Advanced Placement, AP, AP Central, and the acorn logo are
registered trademarks of the College Board. Visit the College Board on the web: collegeboard.org.
AP Central is the ocial online home for the AP Program: apcentral.collegeboard.org.
AP
®
UNITED STATES HISTORY
2019 SCORING GUIDELINES
© 2019 The College Board.
Visit the College Board on the web: collegeboard.org.
Short Answer Question 1
“The revolutionary moment was neither radical nor a watershed for American women. Those who disregard
America’s commitment to patriarchal rule and plead for a historical interpretation that favors enlightened
exceptionalism have overlooked the conditions that made large-scale change all but impossible at that time
and place.”
Elaine Forman Crane, historian, Ebb Tide in New England: Women, Seaports, and Social Change, 16301800,
published in 1998
“The coming of the American Revolution . . . created new opportunities for women to participate in politics.
Responding to men’s appeals, women engaged in a variety of actions in support of the revolutionary cause,
which led women to experience a greater sense of connection to and involvement with the polity. After the war
their political contributions were praised, celebrated, and remembered. . . .Women now were seen as political
beings who had the capacity to influence the course of war, politics, and history.”
Rosemarie Zagarri, historian, Revolutionary Backlash: Women and Politics in the Early American Republic,
published in 2007
Using the excerpts above, answer (a), (b), and (c).
a) Briefly describe ONE major difference between Crane’s and Zagarri’s historical interpretations of the
immediate impact of the American Revolution on women.
b) Briefly explain how ONE event, development, or circumstance from the period 1765 to 1800 that is not
explicitly mentioned in the excerpts could be used to support Crane’s argument.
c) Briefly explain how ONE event, development, or circumstance from the period 1765 to 1800 that is not
explicitly mentioned in the excerpts could be used to support Zagarri’s argument.
Scoring Guide
0–3 points
Score 3
Response accomplishes all three tasks set by the question.
Score 2
Response accomplishes two of the tasks set by the question.
Score 1
Response accomplishes one of the tasks set by the question.
Score 0
Response accomplishes none of the tasks set by the question.
Score NR
No response. Response is completely blank.
AP
®
UNITED STATES HISTORY
2019 SCORING GUIDELINES
© 2019 The College Board.
Visit the College Board on the web: collegeboard.org.
Short Answer Question 1 (continued)
Question-Specific Scoring Guide
ONE point for describing an important difference between Crane’s and Zagarri’s historical
interpretations of the immediate impact of the American Revolution on women
ONE point for explaining how ONE event, development, or circumstance from the period 1765 to 1800
that is not explicitly mentioned in the excerpts could be used to support Crane’s argument
ONE point for explaining how ONE event, development, or circumstance from the period 1765 to 1800
that is not explicitly mentioned in the excerpts could be used to support Zagarri’s argument
Scoring Notes
Introductory notes:
Each point is earned independently.
Accuracy: These rubrics require that students demonstrate historically defensible content knowledge.
Given the timed nature of the exam, responses may contain errors that do not detract from their overall
quality, as long as the historical content used to advance the argument is accurate.
Clarity: Exam responses should be considered first drafts and thus may contain grammatical errors.
Those errors will not be counted against a student unless they obscure the successful demonstration of
the content knowledge, skills, and practices described below.
Examples of responses to (a) that would earn credit:
Crane argues that:
o The American Revolution was not radical because it didn’t change women’s status in society.
o Gender roles for women remained unchanged after the American Revolution.
o The ideals of the American Revolution were not applied to women.
Zagarri argues that:
o Women’s social status and social influence improved after the American Revolution.
o Women were newly able to participate in the country’s politics.
o Women were more visible in civic life/public sphere/society.
NOTE: Credited responses for (a) must explicitly address the substance of both excerpts.
Examples of responses to (b) that would earn credit:
Women could not vote (except in New Jersey, which also eventually rescinded this right).
Women were excluded from the rights stated for all men in the Declaration of Independence.
Women could not participate in any of the political decision-making after the American Revolution.
Abigail Adams asked her husband John Adams to make sure that men remember the ladies,but
women were not involved in political decision-making.
The idea of republican motherhood gave women authority in the private sphere but kept them confined
to the private sphere.
Married women did not have any recognized legal or property rights separate from their husbands.
Women were not allowed to fight in the Continental Army adhering to gender roles.
AP
®
UNITED STATES HISTORY
2019 SCORING GUIDELINES
© 2019 The College Board.
Visit the College Board on the web: collegeboard.org.
Short Answer Question 1 (continued)
Examples of responses to (c) that would earn credit:
Women participated in Revolutionary protest (e.g., boycotts, protests, crowds, homespun movement,
nonconsumption, Daughters of Liberty, Edenton tea party, Ladies of Philadelphia).
Women supported the Revolutionary effort (e.g., fundraising, camp followers, nurses, spies, cooks,
Phillis Wheatley, Deborah Sampson, Molly Pitcher).
Women such as Abigail Adams acted as informal advisors to presidents and political leaders.
The idea of republican motherhood gave women a voice in the shaping of future republican citizens
and an indirect role in public life.
Women applied Revolutionary principles to post-Revolutionary United States society (e.g., Elizabeth
Freeman (aka Mumbet)).
Mercy Otis Warren evoked patriotism in published works on political subjects.
Judith Sargent Murray argued for women’s equality and advocated for women’s education beyond
domestic tasks.
Academies for women offered new educational opportunities.
Women were able to vote in New Jersey between the late 1790s and early 1800s.
Divorce laws in some states were liberalized after the American Revolution.
After the Revolution, women were more apt to obtain custody of their children after divorce if the
children were young.
NOTE: It would be acceptable for test takers to use some of the same examples to respond to both (b) and (c) as
long as the evidence is appropriate for each part of the question. Merely mentioning development or circumstance
will not result in the awarding of points in parts (b) and (c). Responses must specifically explain how the events
represent evidence that can support the author’s claim.
AP
®
UNITED STATES HISTORY
2019 SCORING GUIDELINES
© 2019 The College Board.
Visit the College Board on the web: collegeboard.org.
Short Answer Question 2
Using the image above, which depicts the first half of the nineteenth century, answer (a), (b), and (c).
a) Briefly describe ONE historical perspective expressed in the image.
b) Briefly explain how ONE specific event or development in the period from 1800 to 1850 contributed to the
process depicted in the image.
c) Briefly explain ONE specific historical effect in the period from 1844 to 1890 that resulted from the process
depicted in the image.
Scoring Guide
0–3 points
Score 3
Response accomplishes all three tasks set by the question.
Score 2
Response accomplishes two of the tasks set by the question.
Score 1
Response accomplishes one of the tasks set by the question.
AP
®
UNITED STATES HISTORY
2019 SCORING GUIDELINES
© 2019 The College Board.
Visit the College Board on the web: collegeboard.org.
Short Answer Question 2 (continued)
Score 0
Response accomplishes none of the tasks set by the question.
Score NR
Is completely blank
Question-Specific Scoring Guide
ONE point for describing ONE historical perspective expressed in the image
ONE point for explaining how ONE specific event or development in the period from 1800 to 1850
contributed to the process depicted in the image
ONE point for explaining ONE specific historical effect in the period from 1844 to 1890 that resulted
from the process depicted in the image
Scoring Notes
Introductory notes:
Each point is earned independently.
Accuracy: These rubrics require that students demonstrate historically defensible content knowledge.
Given the timed nature of the exam, responses may contain errors that do not detract from their overall
quality, as long as the historical content used to advance the argument is accurate.
Clarity: Exam responses should be considered first drafts and thus may contain grammatical errors.
Those errors will not be counted against a student unless they obscure the successful demonstration of
the content knowledge, skills, and practices described below.
Examples of responses to (a) that would earn credit:
Artist supports Manifest Destiny and the spread of United States settlers westward; image
suggests that American civilization is spreading.
Image in upper left suggests explorers discovering land that appears to be unoccupied.
The image of the wagon train in the center suggests progress and movement of people westward.
The image may connote that homesteading and establishing new farms will improve the West.
Depiction of American Indians suggests that they are passive observers of United States westward
expansion/accepting of United States expansion.
The depiction of the sun suggests the light of progress through United States westward expansion
(sun is either rising or setting).
The image in the lower right depicts people digging for gold in California, which suggests the
many ways to profit from westward movements (e.g., fur trapping, cattle ranching, mining,
farming, commercial shipping, trade).
All of these views associate Manifest Destiny and the western expansion of the United States with
economic progress, social and geographic mobility, religious mission, and ideas of racial
superiority.
These views also associate Manifest Destiny with white male settlers.
AP
®
UNITED STATES HISTORY
2019 SCORING GUIDELINES
© 2019 The College Board.
Visit the College Board on the web: collegeboard.org.
Short Answer Question 2 (continued)
Examples of responses to (b) that would earn credit:
The Louisiana Purchase added to the territory of the United States that was accessible for national
expansion.
The War of 1812 and the defeat of Tecumseh’s Indian Confederacy opened the Old Northwest to
United States settlement.
Promotion of the idea of opportunities in the West through the press and by journalists (e.g., John
L. O’Sullivan, Horace Greeley) encouraged Americans to move west.
MexicanAmerican War and the resulting acquisition of new land by the United States opened new
land for United States settlement.
Particular population movements into the West contributed to or facilitated westward mobility.
o Migration of family farmers for economic opportunity
o Emergence of new cities and towns in the West
o Oregon Trail allowing for westward movement
o Mormon migration
o Free-soilers moving to the West
Expanded transportation networks (e.g., canals and railroads) facilitated westward expansion.
Economic development of the West encouraged westward migration.
o Agricultural staple production in the Midwest
o Cotton production and slavery expanding to the West
o Growth of the internal slave trade
o Fur trade in the Pacific Northwest
o Development of extractive industries such as mining
o Gold rushes in California and elsewhere
o Development of cattle ranching
American Indian removal led to the dispossession of land that could then be acquired by U.S.
western migrants.
Examples of responses to (c) that would earn credit:
MexicanAmerican War and the resulting acquisition of new land for the United States.
Controversies over the spread of slavery to the West, which ultimately led to the Civil War.
o Kansas–Nebraska Act
o Compromise of 1850
o Free Soil Party
o Foundation of the Republican Party
Developments that contributed to the continuation of population movement into the West between
1844 and 1890.
o Migration of family farmers for economic opportunity
o Emergence of new cities and towns in the West (e.g., Chicago)
o African American “Exoduster” migration from the South
Expanded transportation networks and greater integration of the West with the economies of the
Northeast and Midwest (e.g., transcontinental railroads).
Continued economic development of the West.
o Farming on the Great Plains
o Largely extractive industries such as mining
o Gold rushes in California and elsewhere
o Cattle ranching
Federal government encouragement of western development (e.g., Homestead Act, railroad land
grants, and subsidies).
AP
®
UNITED STATES HISTORY
2019 SCORING GUIDELINES
© 2019 The College Board.
Visit the College Board on the web: collegeboard.org.
Short Answer Question 2 (continued)
United States westward expansion created conflicts between white settlers and American Indians.
o Wars between federal government and American Indians (e.g., Sand Creek massacre
(1864), General George Custer’s campaigns, Battle of Little Big Horn (1876), Dawes Act
(1887), Wounded Knee (1890))
o Forcing of American Indians onto reservations
More interest by business leaders in gaining access to and control over external markets in Latin
America and beyond.
Destruction of American bison (buffalo) and reduction in other natural resources in the West.
International migrants from places such as eastern Asia came to the western United States for
labor and economic opportunities.
The desire of white Americans to exclude nonwhite immigrants from competing for economic
opportunities in the West led to immigration restrictions such as the Chinese Exclusion Act (1882).
Western Agrarian activists (e.g., Grangers, Farmer’s Alliance/Populists) organized to oppose the
monopolistic practices of western transportation companies and agribusinesses.
NOTE: Be sure to note the overlapping dates on (b) and (c) and make sure the answers for (b) and (c) are within the
correct time period specified.
NOTE: It may be acceptable for test takers to use some of the same examples to respond to either (b) or (c) as long
as they show how the evidence is appropriate for each part of the question. Merely mentioning an event will not
result in the awarding of points in part (b) and (c). Responses must provide an explanation that answers the
relevant question.
AP
®
UNITED STATES HISTORY
2019 SCORING GUIDELINES
© 2019 The College Board.
Visit the College Board on the web: collegeboard.org.
Short Answer Question 3
Answer (a), (b), and (c). Confine your response to the period from 1607 to 1754.
a) Briefly describe one difference between the economy of British North American colonies in the Chesapeake
region (such as Virginia and Maryland) and the economy of the middle colonies (such as Pennsylvania and
New York).
b) Briefly describe one similarity between the economy of the Chesapeake colonies and the economy of the
middle colonies.
c) Briefly explain one reason for a difference between the economy of the Chesapeake colonies and the
economy of the middle colonies.
Scoring Guide
0–3 points
Score 3
Response accomplishes all three tasks set by the question.
Score 2
Response accomplishes two of the tasks set by the question.
Score 1
Response accomplishes one of the tasks set by the question.
Score 0
Response accomplishes none of the tasks set by the question.
Score NR
Is completely blank
Question-Specific Scoring Guide
ONE point for describing one difference between the economy of British North American colonies in
the Chesapeake region (such as Virginia and Maryland) and the economy of the middle colonies (such
as Pennsylvania and New York)
ONE point for describing one similarity between the economy of the Chesapeake colonies and the
economy of the middle colonies
ONE point for explaining one reason for a difference between the economy of the Chesapeake colonies
and the economy of the middle colonies
AP
®
UNITED STATES HISTORY
2019 SCORING GUIDELINES
© 2019 The College Board.
Visit the College Board on the web: collegeboard.org.
Short Answer Question 3 (continued)
Scoring Notes
Introductory notes:
Each point is earned independently.
Accuracy: These rubrics require that students demonstrate historically defensible content knowledge.
Given the timed nature of the exam, responses may contain errors that do not detract from their overall
quality, as long as the historical content used to advance the argument is accurate.
Clarity: Exam responses should be considered first drafts and thus may contain grammatical errors.
Those errors will not be counted against a student unless they obscure the successful demonstration of
the content knowledge, skills, and practices described below.
Examples of responses to (a) that would earn credit:
Cash crops from the Chesapeake (such as tobacco) were typically shipped to England, while staple
crops from the middle colonies (such as wheat) were typically shipped to the Caribbean.
The fur trade more extensive in the middle colonies and less extensive in the Chesapeake.
There was more diversity of trade in middle colonies (e.g., furs, various agricultural products), while
trade was more narrowly focused on cash crops such as tobacco in the Chesapeake.
The Chesapeake was more dependent upon slave plantation agriculture than the middle colonies.
The middle colonies were more urban and had more port cities, while the Chesapeake was more rural
and had fewer and smaller urban centers.
The middle colonies were societies with slaves (i.e., not socially dominated by slavery), versus the
Chesapeake, which was a slave society (i.e., socially organized around the principles of chattel
slavery).
Shipbuilding and lumbering were more prevalent industries in the middle colonies than they were in
the Chesapeake region, which focused on cash crop agriculture.
NOTE: Credited responses for (a) must address explicit differences between the Chesapeake and middle colonies.
(e.g., A response like The Chesapeake colonies had fertile soil and the middle colonies did notis not sufficient to
address the prompt.)
NOTE: In the Chesapeake colonies, the use of cotton as the only cash crop will not be accepted; however, tobacco
with cotton will be considered as a minor error and can be considered for credit.
NOTE: The concepts of industrialization and the textile industry should not be accepted. The use of the term
manufacturing must be used in conjunction with other activities that colonies were engaged in economically (e.g.,
farming, shipbuilding, lumbering, and fishing).
NOTE: The use of the geographical terms “the North” and “the South” can be used as long as additional specific
information is given in the response to reference the middle or Chesapeake colonies.
AP
®
UNITED STATES HISTORY
2019 SCORING GUIDELINES
© 2019 The College Board.
Visit the College Board on the web: collegeboard.org.
Short Answer Question 3 (continued)
Examples of responses to (b) that would earn credit:
Both the middle colonies and the Chesapeake engaged in export trade/the triangular trade.
Both the middle colonies and the Chesapeake were part of a mercantilist system centered on Great
Britain.
Both the middle colonies and the Chesapeake exported raw goods to/imported finished goods from
Great Britain.
Both the economies of the middle colonies and Chesapeake were based on agricultural production for
export.
Both the middle and Chesapeake colonies had to pay taxes to England (the response must mention
either the Navigation Acts or the mercantilist system).
Examples of responses to (c) that would earn credit:
Waterways in the Chesapeake were suitable for transport of cash crops produced closer to the coast,
while waterways in the middle colonies (e.g., the Hudson, Susquehanna, and Delaware Rivers), which
included New Netherlands, were more readily navigable and went deeper into the interior of North
American, facilitating regional commerce and the fur trade with Native Americans.
The climate of the Chesapeake region was suitable for valuable cash crops that required a longer
growing season as opposed to the cooler climate of the middle colonies that allowed for grain crops to
be grown.
Development of large-scale slavery in the Chesapeake facilitated cash crop production, as opposed to
the smaller scale of slavery and a greater dependency on indentured servants and immigrant labor in
the middle colonies.
Virginia was founded by joint-stock company investors seeking profit on investments, while the middle
colonies were founded by more diverse European settlers, including religious refugees and individuals
seeking improved economic autonomy and opportunities.
NOTE: Earning the point for (c) is not contingent on earning the point for (a).
NOTE: A credited response to (c) could be a reason for a difference established in (a). If the response in (c) is a
continuation that addresses the same explicit difference established in (a), the response can earn credit if it only
addresses one region.
AP
®
UNITED STATES HISTORY
2019 SCORING GUIDELINES
© 2019 The College Board.
Visit the College Board on the web: collegeboard.org.
Short Answer Question 4
Answer (a), (b), and (c).
a) Briefly describe ONE similarity between New Deal and Great Society programs.
b) Briefly describe ONE difference between New Deal and Great Society programs.
c) Briefly explain ONE reason for a difference between New Deal and Great Society programs.
Scoring Guide
0–3 points
Score 3
Response accomplishes all three tasks set by the question.
Score 2
Response accomplishes two of the tasks set by the question.
Score 1
Response accomplishes one of the tasks set by the question.
Score 0
Response accomplishes none of the tasks set by the question.
Score NR
Is completely blank
Question-Specific Scoring Guide
ONE point for describing ONE similarity between New Deal and Great Society programs
ONE point for describing ONE difference between New Deal and Great Society programs
ONE point for explaining ONE reason for a difference between New Deal and Great Society programs
Scoring Notes
Introductory notes:
Each point is earned independently.
Accuracy: These rubrics require that students demonstrate historically defensible content knowledge.
Given the timed nature of the exam, responses may contain errors that do not detract from their overall
quality, as long as the historical content used to advance the argument is accurate.
Clarity: Exam responses should be considered first drafts and thus may contain grammatical errors.
Those errors will not be counted against a student unless they obscure the successful demonstration of
the content knowledge, skills, and practices described below.
AP
®
UNITED STATES HISTORY
2019 SCORING GUIDELINES
© 2019 The College Board.
Visit the College Board on the web: collegeboard.org.
Short Answer Question 4 (continued)
Examples of responses to (a) that would earn credit:
Both expanded a social welfare net.
Both expanded government programs.
Both regulated society and economy, expanded the power and reach of the federal government, and
increased government spending and the management of fiscal and monetary policy to ensure economic
productivity and growth.
Both addressed poverty and unemployment.
Both had programs to help the elderly.
Both gave workers and labor purchasing power and a stronger political voice.
Both were driven by presidential vision and expanded presidential power.
NOTE: Credited responses for (a) must not rely on generalities. “Both addressed the economy” would not earn a
point but “both addressed poverty and unemployment” would because it addresses specific economic concerns.
Examples of responses to (b) that would earn credit:
The New Deal stabilized capitalism and sought to stave off the deepening or return of the Great
Depression, while the Great Society sought to extend the benefits of affluence to all Americans and to
eradicate poverty.
The New Deal focused on economic recovery and relief, while the Great Society sought to eradicate
social problems like poverty, racial inequality, access to health care, and access to education.
The Great Society focused on civil rights, while much of the New Deal did not address civil rights or
institutionalized discrimination against African Americans and women in its programs.
Great Society environmental policies focused primarily on the beautification of urban and rural areas,
as well as developing regulations for air and water; New Deal environmental policies, instead, focused
more on constructing infrastructure such as roads, bridges, and buildings.
The New Deal created a lasting Democratic political coalition and consensus around liberalism in
government, while aspects of the Great Society hurt the Democratic Party and inspired criticisms of
liberalism by the New Right.
NOTE: Credited responses for (b) must address explicit differences between the New Deal and the Great Society.
(e.g., A response like ‘The Great Society addressed civil rights issues, but the New Deal did not” is not sufficient to
address the prompt.)
NOTE: Credited responses for (b) must elaborate beyond chronological differences.
AP
®
UNITED STATES HISTORY
2019 SCORING GUIDELINES
© 2019 The College Board.
Visit the College Board on the web: collegeboard.org.
Short Answer Question 4 (continued)
Examples of responses to (c) that would earn credit:
The New Deal was a response to an economic depression, while the Great Society was a response to
the persistent problem of poverty in a prosperous economy.
The Great Society took place during and in response to the civil rights movement, while the New Deal
reflected the racism of the era prior to the civil rights movement in American politics.
The New Deal political coalition included many southern Democrats who supported racial
discrimination (although it did feature incremental symbolic progress on racial issues), while the Great
Society response to pressure from civil rights activists led many southerners to leave the Democratic
Party.
The Great Society could build upon the accomplishments of the New Deal, whereas the intervention of
the federal government in society and economy during the New Deal was unprecedented.
The Vietnam War constrained the accomplishments of the Great Society, while the New Deal could be
more expansive partly because the United States was uninvolved in significant international
engagements at the time.
NOTE: Earning the point for (c) is not contingent on earning the point for (b).
NOTE: A credited response to (c) could be a reason for a difference established in (b). If the response in (c) is a
continuation that addresses the same explicit difference established in (b), the response can earn credit if it only
addresses one topic.
AP
®
UNITED STATES HISTORY
2019 SCORING GUIDELINES
© 2019 The College Board.
Visit the College Board on the web: collegeboard.org.
Question 1 Document-Based Question
Evaluate the extent to which the Progressive movement fostered political change in the United States from
1890 to 1920.
Maximum Possible Points: 7
Points
Rubric
Notes
A: Thesis/Claim (01)
Responds to the prompt with a
historically defensible thesis/claim that
establishes a line of reasoning. (1 point)
To earn this point, the thesis must make
a claim that responds to the prompt
rather than restating or rephrasing the
prompt. The thesis must consist of one or
more sentences located in one place,
either in the introduction or the
conclusion.
The thesis must make a historically defensible claim
that establishes a line of reasoning about how the
Progressive movement fostered political change in the
United States from 1890 to 1920.
Examples that earn this point include:
“The Progressive movement was incredibly
successful in fostering political change such as
trust busting large monopolies and reforming the
criminal justice system for youth and adults;
however, their failure to advance African
American civil rights sufficiently left a significant
scar on their legacy, as many issues of
discrimination such as segregation and Jim Crow
were prolonged.
The Progressive Movement in the United States
from 1890 to 1920 fostered great political change,
such as rooting out corruption in government,
eliminating monopolies in business, and by
advocating rights for those who had been
discriminated against.
“The Progressive movement fostered political
change in the United States because it sought for
a deeper respect for democracy, equal and
equitable rights for all people, and it applied new
ways of thinking to services within America.”
B: Contextualization
(01)
Describes a broader historical context
relevant to the prompt.
(1 point)
To earn this point, the response must
relate the topic of the prompt to broader
historical events, developments, or
processes that occur before, during, or
continue after the time frame of the
question. This point is not awarded for
merely a phrase or reference.
To earn the point, the response must accurately
describe a context relevant to how the Progressive
movement fostered political change in the United States
from 1890 to 1920.
Examples of context might include the following, with
appropriate elaboration:
The expansion of industrialization
The growth of cities
The development of large-scale immigration from
southern and eastern Europe
The consolidation of corporations into large trusts
The government’s adherence to laissez-faire
economics
AP
®
UNITED STATES HISTORY
2019 SCORING GUIDELINES
© 2019 The College Board.
Visit the College Board on the web: collegeboard.org.
Question 1 Document-Based Question (continued)
C: Evidence (03)
Evidence from the Documents:
Uses the content of at least three
documents to address the topic of the
prompt. (1 point)
To earn 1 point, the response must
accurately describerather than simply
quotethe content from at least three of
the documents.
OR
Supports an argument in response to
the prompt using at least six
documents. (2 points)
To earn 2 points, the response must
accurately describerather than simply
quotethe content from at least six
documents. In addition, the response
must use the content from the documents
to support an argument in response to the
prompt.
See document summaries page for details.
Doc 1: Jane Addams, Twenty Years at Hull-House,
1910
Doc 2: Theodore Roosevelt, speech, 1902
Doc 3: Julian W. Mack, “The Juvenile Court,” 1909
Doc 4: Hiram Johnson, inaugural address, 1911
Doc 5: NAACP, letter to Woodrow Wilson, 1913
Doc 6: James Couzens, “The Detroit Police
Department,” 1917
Doc 7: Anti-Saloon League cartoon, circa 1918
Evidence Beyond the Documents:
Uses at least one additional piece of
specific historical evidence (beyond that
found in the documents) relevant to an
argument about the prompt.
(1 point)
To earn this point, the evidence must be
described and must be more than a
phrase or reference. This additional piece
of evidence must be different from the
evidence used to earn the point for
contextualization.
Evidence used might include the following, with
appropriate elaboration:
Muckrakers could be used as evidence of the
spread of concern in the United States over the
negative effects of industrialization and the
advocacy for political action to alleviate these
effects.
The Populist (People’s) Party could be used as
evidence of the influence on Progressive thought
of economic reform efforts based in rural areas.
The Federal Reserve Act (1913) could be used as
evidence of the political change made by new
federal government regulation of the national
money supply.
The passage of the Clayton Anti-Trust Act (1914)
could be used as evidence of political change
made by Progressives to counter the reduction in
economic competition caused by corporate
consolidation.
North American Woman Suffrage Association
could be used as evidence of a Progressive reform
effort that changed United States politics by
helping to expand the electorate by pressing for
women’s voting rights.
Eugene V. Debs
W. E. B. Du Bois
Henry Ford
Robert La Follette
AP
®
UNITED STATES HISTORY
2019 SCORING GUIDELINES
© 2019 The College Board.
Visit the College Board on the web: collegeboard.org.
Question 1 Document-Based Question (continued)
D: Analysis and Reasoning (02)
Sourcing:
For at least
three
documents,
explains how or why the document’s
point of view, purpose, historical
situation, and/or audience is relevant to
an argument. (1 point)
To earn this point, the evidence must
explain how or whyrather than simply
identifyingthe document’s point of
view, purpose, historical situation, or
audience is relevant to an argument
about the prompt for each of the three
documents sourced.
See document summaries page for examples of possible
explanations of the relevance of sourcing.
Complexity: Demonstrates a complex
understanding of the historical
development that is the focus of the
prompt, using evidence to corroborate,
qualify, or modify an argument that
addresses the question. (1 point)
A response may demonstrate a complex
understanding in a variety of ways, such
as:
Explaining a nuance of an issue
by analyzing multiple variables
Explaining both similarities and
differences, or explaining both
continuity and change, or
explaining multiple causes, or
explaining both causes and
effects
Explaining relevant and
insightful connections within and
across periods
Confirming the validity of an
argument by corroborating
multiple perspectives across
themes
Qualifying or modifying an
argument by considering diverse
or alternative views or evidence
This understanding must be part of the
argument, not merely a phrase or
reference.
Examples of demonstrating a complex understanding
might include:
Explaining a nuance by exploring the
contradiction that Progressivism supported
greater political participation but also supported
increased government power over people
Explaining similarities and differences in different
Progressive reform movements such as social
reform in cities, economic regulation, and
Prohibition
Explaining connections to other time periods,
such as the reform efforts of the first half of the
nineteenth century
Confirming the validity of the response’s
argument about the greater role of the
government in people’s lives across themes by
explaining how foreign policy in the Progressive
Era involved United States intervention into the
affairs of its new colonies and foreign countries
Qualifying or modifying an argument by
considering evidence that despite the
achievements of Progressive reform, Progressives
contributed to the persistence or even the
expansion of racism and segregation
If response is completely blank, enter - - for all four score categories A, B, C, and D.
AP
®
UNITED STATES HISTORY
2019 SCORING GUIDELINES
© 2019 The College Board.
Visit the College Board on the web: collegeboard.org.
Question 1 Document-Based Question (continued)
Document Summaries and Possible Sourcing
Document
Response explains the relevance of point of view,
purpose, situation, and/or audience by elaborating
on examples such as:
1. Jane
Addams, 1890s
Describes political campaign by
social reformers against a local
official or machine politics
Discovered many voters had jobs
gained through this official or
political machine
Describes expectation that urban
political officials will provide
unofficial services for their
constituents and describes the
surprise of Hull-House residents
that reformers refused to provide
these services, leading to
conflicts
Immigration and industrialization in the late
nineteenth and early twentieth centuries led to
rapid population growth in cities such as Chicago
and an expansion in demand on political leaders
for municipal services (situation).
As a social activist, Addams believed that many of
the services provided to urban immigrant
communities by political leaders represented
corruption to be fought against and stopped by
political reform movements (point of view).
Progressive women reformers like Addams and
other workers at Hull-House engaged in political
and social activism and sought to influence
government policy and reduce corruption
(purpose).
In their efforts to reform local politics and political
machines, Addams and other Progressive
reformers undermined the social service and
political patronage system that immigrants relied
upon (purpose).
Progressive reformers like Addams sought to
exercise social control by cleaning up urban
political machines and uplifting immigrants by
Americanizing them (point of view).
2. Theodore
Roosevelt,
1902
Argues that large corporations
(“trusts”) should be regulated by
governments
Describes the existence and
survival of large corporations as
reliant upon an interdependent
relationship with the state.
As a Progressive president, Theodore Roosevelt
trusted government and therefore argued for the
expansion of federal government power over
private enterprise (point of view).
Large-scale corporate consolidation by the early
1900s led to public concern over growth of
corporations and their political influence
(situation).
Roosevelt intends to persuade national audiences
that it is good for the government to expand its
power in order to regulate big business and
corporations (purpose).
Roosevelt’s position in the document raised
questions among Progressives about the proper
role of the government in regulating businesses
and the economy (situation).
AP
®
UNITED STATES HISTORY
2019 SCORING GUIDELINES
© 2019 The College Board.
Visit the College Board on the web: collegeboard.org.
Question 1 Document-Based Question (continued)
3. Julian W.
Mack, “The
Juvenile
Court,” 1909
Argues courts have responsibility
to help juvenile offenders and
neglected children
Argues that it is the duty of the
state to develop citizenship
Progressive reformers argued that it was the duty
of the state to intervene in the lives of some
Americans to address social problems such as
juvenile delinquency (point of view).
The rapid growth of the population of cities such
as Chicago led to the inadequacy of many
municipal services, such as limited educational
and recreational opportunities for young people
(situation).
Mack believed that the state has a role in
protecting children when parents and families fail
to do so, and he advocated for an extension of the
role of government in the private sphere (purpose).
Mack was writing for lawyers and judges in order
to encourage the courts to interpret laws in a way
that gave more power to the state to reform
families (audience).
4. Governor
Hiram
Johnson, 1911
Advocates for initiative,
referendum, and recall
Advocates for popular
participation in government
The growth of large corporations and municipal
governments in the late 1800s and early 1900s
increased the opportunities for political corruption
(situation).
Progressives sought to rally support for state
legislative reforms that empowered citizens
through direct democracy and reduced corruption
and the influence of big business (purpose).
Expand democracy and political participation in
the government in order to stave off more radical
political ideas (purpose).
Johnson sought to appeal to the citizens of
California to support his political and legislative
agenda to rein in corporate power and increase
power to the people (audience).
AP
®
UNITED STATES HISTORY
2019 SCORING GUIDELINES
© 2019 The College Board.
Visit the College Board on the web: collegeboard.org.
Question 1 Document-Based Question (continued)
5. NAACP,
letter to
Woodrow
Wilson, 1913
Protests racial segregation in
federal government employment
under the Wilson administration
Argues that the separate but
equal doctrine is inherently
unequal
As an organization devoted to advocating for the
rights of African Americans, the NAACP opposed
the policies of Woodrow Wilson that increased
racial segregation (point of view).
Jim Crow racial segregation laws spread in the
United States after the Plessy v. Ferguson decision
(1896) with the active support of many Progressive
reformers (situation).
The expanded federal bureaucracy made federal
employment a symbolic bastion of segregation
during the Wilson administration (situation).
African American activists sought to convince
President Wilson to take political action to stop
furthering and justifying racial segregation
(purpose).
6. James
Couzens,
“Detroit Police
Department,”
1917
Promotes efficiency in police
work
Describes reorganizing work of
beat cops and detectives
Responding to public demands
for order
As a former business executive, this government
official argued that scientific and management
principles from business could be applied to
government services and used to solve social
problems (point of view).
New technologies and methods of organizing labor
in industry in the late 1800s and early 1900s led to
increased economic efficiency and growth
(situation).
7. American
Issue
Publishing
Company,
“Wet or Dry”
cartoon, circa
1918
urging people to “Vote wet for my
sake!” with a woman surrounded
by three children urging people
to “Vote dry for mine!”
Text urges voters to “Vote Dry.”
Produced by an organization that opposed the use
of alcohol, the cartoon asserts that the government
has a role in reducing alcohol consumption and
protecting the well-being of women and families
and preventing the spread of social ills associated
with the consumption of alcohol (point of view).
Many Progressive reformers in the early 1900s
urged voters to support a constitutional
amendment to prohibit the production of alcohol
(purpose).
A long-standing temperance movement, often led
by white, Protestant women reformers, reached a
crescendo in the late Progressive Era with its calls
for legislative action by the government (situation).
AP
®
UNITED STATES HISTORY
2019 SCORING GUIDELINES
© 2019 The College Board.
Visit the College Board on the web: collegeboard.org.
Question 1 Document-Based Question (continued)
Scoring Notes
Introductory notes:
Except where otherwise noted, each point of these rubrics is earned independently, e.g., a student
could earn a point for evidence without earning a point for thesis/claim.
Accuracy: The components of these rubrics require that students demonstrate historically defensible
content knowledge. Given the timed nature of the exam, essays may contain errors that do not detract
from their overall quality, as long as the historical content used to advance the argument is accurate.
Clarity: Exam essays should be considered first drafts and thus may contain grammatical errors.
Those errors will not be counted against a student unless they obscure the successful demonstration of
the content knowledge, skills, and practices described below.
Note: Student samples are quoted verbatim and may contain grammatical errors.
A. Thesis/Claim (01 point)
Responses earn 1 point by responding to the prompt with a historically defensible claim that establishes a line
of reasoning about the topic. To earn this point, the thesis must make a claim that responds to the prompt
rather than simply restating or rephrasing the prompt. The thesis must suggest at least one main line of
argument development or establish the analytic categories of the argument.
The thesis must consist of one or more sentences located in one place, either in the introduction or the
conclusion.
Examples of acceptable theses:
“The Progressive movement was incredibly successful in fostering political change such as trust
busting large monopolies and reforming the criminal justice system for youth and adults; however,
their failure to advance African American civil rights sufficiently left a significant scar on their legacy,
as many issues of discrimination such as segregation and Jim Crow were prolonged.” (This example
suggests a historically defensible line of argument development and establishes the main analytical
categories of the response, including a counterargument.)
“The Progressive Movement in the United States from 1890 to 1920 fostered great political change,
such as rooting out corruption in government, eliminating monopolies in business, and by advocating
rights for those who had been discriminated against.” (This example suggests a historically defensible
line of argument development and establishes the main analytical categories of the response.)
“While the Progressive movement achieved goals in the improvement of cities and the protection of
children, the movement ultimately failed in increasing democracy due to the fact that democratic rights
were not extended to African Americans.” (This example suggests a historically defensible line of
argument development and establishes the main analytical categories of the essay.)
“The Progressive movement fostered political change in the United States because it sought for a
deeper respect for democracy, equal and equitable rights for all people, and it applied new ways of
thinking to services within America.” (This example suggests a historically defensible line of argument
development and establishes the main analytical categories of the essay.)
AP
®
UNITED STATES HISTORY
2019 SCORING GUIDELINES
© 2019 The College Board.
Visit the College Board on the web: collegeboard.org.
Question 1 Document-Based Question (continued)
Examples of unacceptable theses:
“The Progressive movement caused political change to a large extent because it influenced so many
different areas of politics that were able to change at the time.” (This example largely restates the
prompt.)
“Political, social, and economic change occurred during the Progressive Era.” (This example is too
vague and does not establish a line of reasoning.)
“During the time period of 1890 to 1920, the Progressive movement fostered political change in the
United States as new ideas were being spread and as a result of new policies being instituted.” (This
example is too vague and does not establish a line of reasoning.)
B. Contextualization (01 point)
Responses earn 1 point by describing a broader historical context relevant to the topic of the prompt. To earn
this point, the response must accurately and explicitly connect the context of the prompt to broader historical
events, developments, or processes that occurred before, during, or continued after the time frame of the
question. This point is not awarded for merely a phrase or reference.
To earn the point, the response must accurately describe a context relevant to how the Progressive movement
fostered political change in the United States from 1890 to 1920.
Examples of context might include:
The expansion of industrialization
The growth of cities
The development of large-scale immigration from southern and eastern Europe
The consolidation of corporations into large trusts
The government adherence to laissez-faire economics
Examples of acceptable contextualization:
“The mid-nineteenth century was a booming time for urbanization, immigration, and industrialization.
Towns and cities started moving skyward, people from Europe came to America in hopes of achieving
the American dream, and industries such as steel and textiles were booming in the wake of a new work
force. The amount and quality of work in America began to highlight deficiencies in America beyond
the workforce. The Progressive movement began in response to this.” (This example describes a
broader historical context relevant to the topic of the prompt.)
“At the time of 1890, the Gilded Age had been going on for approximately 20 years. The Gilded Age
was the time when large corporations and ‘robber barons’ came to power. Trusts such as John D.
Rockefeller’s Standard Oil were in power and they had monopolies, snuffing out smaller competition
under the guise of Social Darwinism, a principle saying only the strong survive in the business world.
This leads into the progressive movement.” (This example describes a broader historical context relevant
to the topic of the prompt.)
AP
®
UNITED STATES HISTORY
2019 SCORING GUIDELINES
© 2019 The College Board.
Visit the College Board on the web: collegeboard.org.
Question 1 Document-Based Question (continued)
Examples of unacceptable contextualization:
“In 1890 the census of 1890 concludes that there is no more American frontier. President McKinley is
finishing the Spanish American War in 1898 when he gets shot and passes the presidency off to
Theodore Roosevelt who helps the country with progressivism.(This example did not earn credit for
contextualization because it is presenting evidence that is not clearly relevant to how the Progressive
movement fostered political change in the United States.)
“During the mid-1800s there was a great divide in the U.S. This divide was between slavery ultimately
it led to a war which in the end damaged the U.S. It not only damaged U.S. morale, but also damaged
many parts of the South and more greatly the economy. The divide was due to many disagreements
which was somewhat warned by Washington who said don’t form political parties because they would
cause a divide among the states.(This example did not earn credit for contextualization because it does
not provide any evidence that is relevant to how the Progressive movement fostered political change in the
United States.)
C. Evidence (0–3 points)
Evidence from the Documents
In order to earn 1 point for using evidence from the documents, the response must address the topic of the
prompt by using at least three documents. To earn 1 point for evidence from the documents, the response must
accurately describerather than simply quote or paraphrasecontent from at least three of the documents
to address the topic of the prompt.
Examples of describing the content of a document:
“Document 6 says that with the introduction of new technology the man power in the police
department can now be used to keeping the city safer which taxpayers hope for.” (This example
describes evidence from the documents relevant to the topic, so it contributes toward the first evidence
point, but it does not use that evidence to support an argument about the extent to which the Progressive
movement caused political change in the United States, so it does not contribute toward the second point.)
“Teddy Roosevelt in Document 2 is saying that the state needs to control these business corporations
that are becoming too powerful. They’re benefitting themselves instead of the workers.” (This example
correctly describes the content of Document 2, but it does not use that evidence to support an argument that
is relevant to the Progressive movement fostering political change in the United States.)
Examples of unacceptably describing the content of a document:
“In the 1890s ‘when [Alderman] protected a lawbreaker from the legal consequences his kindness
appeared […] when Hull House on the other hand insisted that law must be enforced, it could but
appear as the persecution of the offender’ (Doc. 1).”(This example simply quotes from Document 1 and
does not use the content of this document to address the topic of the prompt.)
“Wilson, in Document 5, is telling us that African Americans are being separated from whites in the
work force so they can be protected from discrimination.” (This example is a misunderstanding of
Document 5. The response attributes the document to President Wilson instead of the NAACP and
misunderstands the content of the document.)
AP
®
UNITED STATES HISTORY
2019 SCORING GUIDELINES
© 2019 The College Board.
Visit the College Board on the web: collegeboard.org.
Question 1 Document-Based Question (continued)
OR
Document Content Supporting an Argument
Responses earn 2 points by using the content of at least six documents to support an argument that responds
to the prompt. To earn 2 points, responses must accurately describe the document’s content; they cannot earn
a point by merely quoting or paraphrasing the documents with no connection to the topic of the prompt.
Examples of supporting an argument using the content of a document:
“In a letter to Wilson, the NAACP points out to segregationists that by having segregation, especially
in the Federal government, promotes inequality (Doc. 5). Virtually no political change came about for
black people during this time.” (This example uses Document 5 to support an argument that the
Progressive era did not bring about political change for African Americans.)
“In keeping with this idea of the rights of children, the Progressive movement also aligned with the
temperance movement for the protection of families. The temperance movement aimed at abolishing
alcohol, and was commonly supported by mothers, who saw the detrimental effects of alcohol leading
to the abuse of mothers and children within the family. The political cartoon encouraged voters to ‘Vote
Dry’ in favor of politicians who opposed the consumption of alcohol (Doc. 7).” (This example connects
the content of Document 7 to an argument about how the Progressive movement attempted to protect
families and the rights of children.)
Example of unacceptably supporting an argument using the content of a document:
“The Departments at Washington had segregated employees so the NAACP requested that President
Wilson desegregate them (Doc. 5). Some may argue that because of this segregation, it must have
meant that progressive reform had no impact, but that is not the case because later it would be
required by law to desegregate work places.” (This example describes the content of Document 5 but uses
this document to support an erroneous argument that desegregation in the workplace will eventually be
achieved because of the Progressive Era.)
“Document 7 says that people should vote for prohibition to help the mothers and children in the
country.” (This example is a correct description of Document 7 and would achieve credit for using the
content of a document to address the topic of Progressivism. However, this example does not tie its
observation about Document 7 to an argument about how the Progressive movement fostered political
change in the United States.)
Evidence Beyond the Documents
In order to earn 1 point for evidence beyond the documents, the response must use at least one additional piece
of specific historical evidence (beyond that found in the documents) relevant to an argument that addresses the
topic. To earn this point, the evidence must be described and must be more than a phrase or reference.
This additional piece of evidence must be different from the evidence used to earn the point for
contextualization. Typically, statements credited as contextualization will be more general statements that
place an argument or a significant portion of it in a broader context. Statements credited as supporting
evidence beyond the documents will typically be more specific details that function as support for a
particular point made in an argument, analogous to the function of evidence drawn from the documents.
AP
®
UNITED STATES HISTORY
2019 SCORING GUIDELINES
© 2019 The College Board.
Visit the College Board on the web: collegeboard.org.
Question 1 Document-Based Question (continued)
Evidence used might include the following, with appropriate elaboration:
Muckrakers could be used as evidence of the spread of concern in the United States over the
negative effects of industrialization and the advocacy for political action to alleviate these effects.
The Populist (People’s) Party could be used as evidence of the influence on Progressive thought of
economic reform efforts based in rural areas.
The Federal Reserve Act (1913) could be used as evidence of the political change made by new
federal government regulation of the national money supply.
The passage of the Clayton Anti-Trust Act (1914) could be used as evidence of political change
made by Progressives to counter the reduction in economic competition caused by corporate
consolidation.
The North American Woman Suffrage Association (NAWSA) could be used as evidence of a
Progressive reform effort that changed United States politics by helping to expand the electorate by
pressing for women’s voting rights.
Muckrakers
Populist (People’s) Party
Federal Reserve Act (1913)
Clayton Anti-Trust Act (1914)
North American Woman Suffrage Association
Eugene V. Debs
W. E. B. Du Bois
Henry Ford
Robert La Follette
New Freedom (associated with Woodrow Wilson)
New Nationalism (associated with Theodore Roosevelt)
Prohibition
Sherman Anti-Trust Act (1890)
Upton Sinclair, The Jungle
William Howard Taft
Frederick W. Taylor
Examples of acceptable use of an additional piece of specific historical evidence:
“Another example of successful use of government to enact change was the women’s rights movement,
in which the work of suffragists to gain the 19
th
amendment was clearly more effective for enacting
fully-recognized change than the smaller-scale and less ambitious idea of Republican Motherhood.”
(This example uses a particular piece of evidence beyond the documents to make a connection to a larger
argument.)
“One effective Progressive reform would be that which was started by the book The Jungle. This book
sought to expose the dangerous conditions workers suffered in the meat factories of Chicago and
President Roosevelt immediately took action after the book, calling for legislation to be implemented,
investigations to be done, and the FDA to be establishedall of which would see to more political
power for the Federal government.” (This example uses The Jungle to support an argument about
increasing the regulatory powers of the federal government during the Progressive Era.)
AP
®
UNITED STATES HISTORY
2019 SCORING GUIDELINES
© 2019 The College Board.
Visit the College Board on the web: collegeboard.org.
Question 1 Document-Based Question (continued)
Examples of unacceptable use of an additional piece of specific historical evidence:
“Many organizations formed in response to this [Document 5] and pushed for political change by going
on marches, such as the March on Washington.” (This example would not earn credit because it is
historically inaccurate.)
“They had people begin voting for Senators.” (This example is the start of an acceptable discussion, but
the allusion to the Seventeenth Amendment is no more than a brief reference and is not fully described.)
D. Analysis and Reasoning (02 points)
Document Sourcing
For at least three documents, the response explains how or why the document’s point of view, purpose,
historical situation, and/or audience is relevant to an argument that addresses the prompt. To earn this point,
the response must explain how or whyrather than simply identifyingthe document’s point of view,
purpose, historical situation, or audience is relevant to an argument addressing the prompt for each of the
three documents sourced.
Example of an acceptable explanation of the relevance of the document’s point of view:
“Couzens’ point of view [Doc. 6] comes from the fact that he had been the manager for Ford Motor
Company which created the efficient assembly line, so he would try to implement the success of the
Ford Motor Co. into the police departments.” (This example correctly identifies Couzens’s point of view as
that of a former manager for Ford Motor Company and connects it to an argument about why reformers
would want to improve the efficiency of urban police departments.)
Example of an unacceptable explanation of the relevance of the document’s point of view:
“Addams’ point of view comes as she is the founder of Hull House, illustrating that these were
observations she obtained herself.” (This example does not contribute toward a point for document
sourcing because it inaccurately discusses the relevance of Addams’s point of view.)
Example of an acceptable explanation of the relevance of the document’s purpose:
“Thus, many of these state reforms [Doc. 4] had the purpose of eventually creating national change by
means of say, an amendment such as the 17
th
amendment that gave voters direct election of their
senators, however, national change was not always seen.” (This example contributed toward a point for
document sourcing because it correctly identifies Johnson’s purpose in supporting several political
reforms.)
Example of an unacceptable explanation of the relevance of the document’s purpose:
“This letter [Doc. 5] has the purpose to ask President Woodrow Wilson to change the legislative
policies of the time so that there can be more equal opportunities for Blacks.” (This example does not
contribute toward a point for document sourcing because it only describes the content of Document 5 and is
not adding any insight into the relevance of the NAACP’s purpose in writing this letter to President Wilson.)
Example of an acceptable explanation of the relevance of the historical situation of a document:
“Roosevelt, in Document 2, advocated for much more power for the President to supervise and control
trusts. Roosevelt was able to control the trusts as a result of the Sherman Anti-Trust Act.” (This
example contributes to a point for document sourcing because it links the historical situation of the
Sherman Anti-Trust Act to an argument about Roosevelt increasing the power of the federal government.)
AP
®
UNITED STATES HISTORY
2019 SCORING GUIDELINES
© 2019 The College Board.
Visit the College Board on the web: collegeboard.org.
Question 1 Document-Based Question (continued)
Example of an unacceptable explanation of the relevance of the historical situation of a document:
Document 3 shows how the Progressive movement led to government regulation because at the time
Chicago was a major city with lots of Progressive influence.” (This example does not contribute toward a
point for document sourcing because it does not explain why the fact that Chicago was a major cityis
relevant in understanding the document.)
Example of an acceptable explanation of the relevance of the audience:
“Document 6 says that segregation continued in government and insulted progressive efforts on behalf
of African Americans. It is significant that the NAACP is addressing President Wilson because Wilson
claimed to be a Progressive but they are pointing out that he was harming efforts for improved civil
rights for African Americans.” (This example contributes to a point for document sourcing because it
explains the relevance of the audience of the NAACP’s letter that President Wilson claimed to be a
Progressive.”)
Example of an unacceptable explanation of the relevance of the audience:
“Hiram Johnson’s audience in Doc. 4 was the people of California.” (This example does not contribute to
a point for document sourcing because it just restates information provided about the document and does
not explain how or why this information is relevant to an argument about Progressivism.)
Demonstrating Complex Understanding
The response demonstrates a complex understanding, using evidence to corroborate, qualify, or modify an
argument that addresses the question.
Demonstrating a complex understanding might include:
Explaining a nuance by exploring the contradiction that Progressivism supported greater political
participation but also supported increased government power over people
Explaining similarities and differences in different Progressive reform movements such as social
reform in cities, economic regulation, and Prohibition
Explaining connections to other time periods, such as the reform efforts of the first half of the
nineteenth century
Confirming the validity of the response’s argument about the greater role of the government in people’s
lives across themes by explaining how foreign policy in the Progressive Era involved United States
intervention into the affairs of its new colonies and foreign countries
Qualifying or modifying an argument by considering evidence that shows the limitations on the
Progressive movement in achieving political change
This understanding must be part of the argument, not merely a phrase or reference.
AP
®
UNITED STATES HISTORY
2019 SCORING GUIDELINES
© 2019 The College Board.
Visit the College Board on the web: collegeboard.org.
Question 1 Document-Based Question (continued)
Examples of acceptable demonstration of a complex understanding:
This response uses multiple pieces of evidence to corroborate its argument that the Progressive movement
was both a local and a national movement to fight political corruption. The response uses content from
Documents 3, 4, and 5 to corroborate this claim, and it also weaves in evidence beyond the documents
about muckraking and W. E. B. DuBois to further support this sophisticated argument. This paragraph
begins by saying, “Between 1890 and 1920, there was notable change in the fact that local political
reforms and nationwide social reforms both exposed the corruptness of the political atmosphere, using
propaganda and local campaigns to tackle a nationwide epidemic of political corruption.” The response
then qualifies its argument by stating, “However, there is a greater change in the fact that no local and
social reforms could combat the spoils system and big business’ effect on politics. . . . In Doc. 2,
Roosevelt explains that big corporations need to be controlled and supervised. Roosevelt’s intended
audience was big business to warn them of the fact that they cannot keep control of politics forever,
and the American people to encourage them to have hope that reform will be made. Roosevelt further
developed these ideas when running for the Progressive Bull Moose party in 1912 against Wilson.
Roosevelt’s speech shows that the power of big business over politics was so grand that it needed
control, showing a continuation in their omnipotence in politics despite criticism from progressives.”
Ultimately, this response demonstrates a complex understanding of how the Progressive movement fostered
political change but also the political and economic forces that ultimately limited this change.
This response demonstrates a complex understanding of the Progressive Era by qualifying its argument.
“Lastly, while the progressive movement led to some political achievements in increasing the
democracy of voters, it failed to achieve true protection of democratic principles in that Black voters
were still limited in terms of their voting rights. During the progressive movement, many democratic
ideals were seen at the state level. Most notably was the initiative, referendum, and recall programs
established by many state governments. The response then uses multiple pieces of evidence, such as
Document 4 and a discussion of the 17
th
Amendment, to support the argument that some reforms were
aimed at expanding democratic principles. The response then qualifies this argument by stating,
“However, national change was not always seen. Under the Woodrow Wilson presidency, Woodrow
Wilson would segregate White House offices and would clearly support racism, perhaps best seen
through his viewing of The Birth of a Nation at the White House, a film endorsed by the KKK which
viewed the Civil War as a war for the struggle of white people, not African slaves. In response to these
racist policies, interest groups such as the NAACP called out Wilson for his failure to protect the rights
of black people, despite other Progressive Era reforms already occurring (Doc. 5). . . . The NAACP was
frustrated with America, after all, the 13
th
, 14
th
, and 15
th
Amendments which aimed at abolishing
slavery, granting equal protection under the law and granting all blacks the right to vote, were not
protecting people in the shadow of sharecropping, segregation, and Jim Crow laws.” The entire
response is rich with evidence and demonstrates a complex understanding of the Progressive Era. This
lengthy paragraph, in particular, offers a sophisticated qualification to its argument.
AP
®
UNITED STATES HISTORY
2019 SCORING GUIDELINES
© 2019 The College Board.
Visit the College Board on the web: collegeboard.org.
Question 1 Document-Based Question (continued)
Example of unacceptably demonstrating complex understanding:
The following response attempts to explain relevant and insightful connections within and across time
periods, but it does not quite demonstrate a complex understanding of the Progressive Era nor does it
clearly use evidence to corroborate or qualify its argument. “While progressives accomplished and
bettered American society overall, the long-term impacts on the African American community and our
failure to learn from mistakes of the Anti-Saloon League and Dry Movement plague us today. Ending
discriminatory practices would have made Black Americans’ situations better in the long run but that
did not happen. Legislators chose to enact the War on Drugs,showing how we were unable to learn
from the Progressives’ failures in reducing trafficking and drug crime. The importance of the
Progressive Era cannot be understated, but we must remain objective in our assessments, criticizing
the negatives as well.” Ultimately, while this is a noble attempt at demonstrating a complex understanding
of the Progressive Era, the response is somewhat simplistic, and the references to modern society are not
fully developed.
AP
®
UNITED STATES HISTORY
2019 SCORING GUIDELINES
© 2019 The College Board.
Visit the College Board on the web: collegeboard.org.
Question 2 — Long Essay Question
Evaluate the extent to which ideas of self-government influenced American colonial reaction to British imperial
authority in the period from 1754 to 1776.
Maximum Possible Points: 6
Points
Rubric
Notes
A: Thesis/Claim
(01)
Thesis/Claim: Responds to the prompt with
a historically defensible thesis/claim that
establishes a line of reasoning. (1 point)
To earn this point, the thesis must make a
claim that responds to the prompt rather than
restating or rephrasing the prompt. The thesis
must consist of one or more sentences located
in one place, either in the introduction or the
conclusion.
The thesis must make a historically defensible
claim that establishes a line of reasoning about how
ideas of self-government influenced American
colonial reaction to British imperial authority from
1754 to 1776.
Examples that earn this point include:
“Although these taxes are considered to be
the reasoning behind the American
Revolution, the impending war was more so
about the need for individual freedom and
republican government that the British
deprived them of after the end to salutary
neglect.”
“During the era of the Enlightenment,
colonists clung to these ideas of
Republicanism and self-government and
used them to justify opposition to Britain’s
economic control and political control of the
colonies.”
B: Contextualization
(01)
Contextualization: Describes a broader
historical context relevant to the prompt.
(1 point)
To earn this point, the response must relate the
topic of the prompt to broader historical
events, developments, or processes that occur
before, during, or continue after the time frame
of the question. This point is not awarded for
merely a phrase or a reference.
To earn the point, the response must accurately
describe a context relevant to how ideas of self-
government influenced American colonial reaction
to British imperial authority in the period from 1754
to 1776.
Examples of context might include the following, with
appropriate elaboration:
Mayflower Compact and/or
acknowledgment of first chartered colonies
Enlightenment ideas about liberty and
consent in government
Colonial experience with local control of
religious institutions (e.g.,
Congregationalist church)
Salutary neglect / Limited enforcement of
the Navigation Acts (1651, 1660, and 1663)
AP
®
UNITED STATES HISTORY
2019 SCORING GUIDELINES
© 2019 The College Board.
Visit the College Board on the web: collegeboard.org.
Question 2 Long Essay Question (continued)
C: Evidence (02)
Evidence:
Provides specific examples of
evidence relevant to the topic of the prompt.
(1 point)
To earn the first point, the response must
identify specific historical examples of
evidence relevant to the topic of the prompt.
OR
Supports an Argument: Supports an
argument in response to the prompt using
specific and relevant examples of evidence.
(2 points)
To earn the second point, the response must
use specific historical evidence to support an
argument in response to the prompt.
Examples of evidence used might include:
Virginia House of Burgesses
Mayflower Compact
Albany Congress (1754)
Seven Years’ War (French and Indian War),
17541763
Pontiac’s Rebellion (1763)
Royal Proclamation of 1763
Virtual representation
Sons of Liberty (Massachusetts)
Nonimportation/nonconsumption
Quartering Act (1765)
Vice admiralty courts
Stamp Act and protests (17651766)
Stamp Act Congress
“No taxation without representation”
Townshend Act (1767)
Boston Massacre (1770)
Tea Act (1773)
Boston Tea Party (1773)
Coercive/Intolerable Acts (1773)
First and Second Continental Congress
Olive Branch Petition (1775)
Thomas Paine, Common Sense (1776)
Declaration of Independence (1776)
AP
®
UNITED STATES HISTORY
2019 SCORING GUIDELINES
© 2019 The College Board.
Visit the College Board on the web: collegeboard.org.
Question 2 Long Essay Question (continued)
D: Analysis and Reasoning (02)
Historical Reasoning: Uses historical
reasoning (e.g., comparison, causation,
continuity and change over time) to frame or
structure an argument that addresses the
prompt. (1 point)
To earn the first point, the response must
demonstrate the use of historical reasoning to
frame or structure an argument, although the
reasoning might be uneven or imbalanced.
OR
Complexity: Demonstrates a complex
understanding of the historical development
that is the focus of the prompt, using
evidence to corroborate, qualify, or modify
an argument that addresses the question.
(2 points)
To earn the second point, the response must
demonstrate a complex understanding. This
can be accomplished in a variety of ways, such
as:
Explaining a nuance of an issue by
analyzing multiple variables
Explaining both similarity and difference,
or explaining both continuity and change,
or explaining multiple causes, or
explaining both causes and effects
Explaining relevant and insightful
connections within and across periods
Confirming the validity of an argument by
corroborating multiple perspectives across
themes
Qualifying or modifying an argument by
considering diverse or alternative views or
evidence
This understanding must be part of the
argument, not merely a phrase or reference.
Examples of using historical reasoning to frame or
structure an argument might include:
Explaining how particular ideas from the
Enlightenment helped cause American
resistance to British rule, such as the Stamp
Act protests or the Boston Tea Party
Tracing the continuity of earlier institutions
of self-government such as the Virginia
House of Burgesses to the role of local
governing bodies in colonial protests, such
as committees of correspondence
OR
Ways of demonstrating a complex understanding of
this prompt might include the following:
Explaining a nuance of an issue by
analyzing multiple variables by, for
example, assessing how ideas of self-
government played out in different colonies,
such as Virginia and Massachusetts
Explaining multiple causes of American
resistance to British colonial authority, such
as the development of a unique American
identity
Explaining relevant and insightful
connections within and across periods by,
for example, making comparisons with
arguments over federalism in the 1780s and
1790s or states’ rights in the 1850s or 1960s
Confirming the validity of an argument by
corroborating multiple perspectives across
themes by, for example, exploring cultural
ideas about British American
exceptionalism or the socioeconomic
differences of British American societies
from Great Britain
Qualifying or modifying an argument by
considering diverse or alternative views or
evidence by, for example, examining the
reasons why colonial resistance against the
Navigation Acts before the 1760s was
limited compared to the colonial reaction to
British imperial authority after 1763
If response is completely blank, enter - - for all four score categories: A, B, C, and D.
AP
®
UNITED STATES HISTORY
2019 SCORING GUIDELINES
© 2019 The College Board.
Visit the College Board on the web: collegeboard.org.
Question 2 Long Essay Question (continued)
Scoring Notes
Introductory notes:
Except where otherwise noted, each point of these rubrics is earned independently, e.g., a student
could earn a point for evidence without earning a point for thesis/claim.
Accuracy: The components of these rubrics require that students demonstrate historically defensible
content knowledge. Given the timed nature of the exam, essays may contain errors that do not detract
from their overall quality, as long as the historical content used to advance the argument is accurate.
Clarity: Exam essays should be considered first drafts and thus may contain grammatical errors.
Those errors will not be counted against a student unless they obscure the successful demonstration of
the content knowledge, skills, and practices described below.
Note: Student samples are quoted verbatim and may contain grammatical errors.
A. Thesis/Claim (01 point)
Responses earn 1 point by responding to the prompt with a historically defensible claim that establishes a line
of reasoning about the topic. To earn this point, the thesis must make a claim that responds to the prompt
rather than simply restating or rephrasing the prompt. The thesis must suggest at least one main line of
argument development or establish the analytic categories of the argument.
The thesis must consist of one or more sentences located in one place, either in the introduction or the
conclusion.
Examples of acceptable theses:
“Although these taxes are considered to be the reasoning behind the American Revolution, the
impending war was more so about the need for individual freedom and republican government that the
British deprived them of after the end to salutary neglect.” (The response makes a historically defensible
claim that addresses the prompt and establishes a line of reasoning.)
“During the era of the Enlightenment, colonists clung to these ideas of Republicanism and self-
government and used them to justify opposition to Britain’s economic control and political control of
the colonies.” (The response makes a historically defensible claim that addresses the prompt and
establishes a line of reasoning.)
Examples of unacceptable theses:
The colonists of North America, armed with their new ideas of a self-ruling government, paved the
way for a new country.” (This example is too vague and does not directly respond to the prompt.)
“Ideas of self-government significantly influenced American colonial reaction to British imperial
authority.” (The response simply restates the prompt with an additional adverb and does not make a
historically defensible claim.)
The unfair expectations from the British ultimately led to the American Revolution.” (This thesis
attempt does not respond to the prompt.)
AP
®
UNITED STATES HISTORY
2019 SCORING GUIDELINES
© 2019 The College Board.
Visit the College Board on the web: collegeboard.org.
Question 2 Long Essay Question (continued)
B. Contextualization (01 point)
Responses earn 1 point by describing a broader historical context relevant to the topic of the prompt. To earn
this point, the response must accurately and explicitly connect the context of the prompt to broader historical
events, developments, or processes that occurred before, during, or continued after the time frame of the
question. This point is not awarded for merely a phrase or reference.
To earn the point, the response must accurately describe a context relevant to how ideas of self-government
influenced American colonial reaction to British imperial authority from 1754 to 1776.
Examples might include the following, with appropriate elaboration:
Mayflower Compact and/or acknowledgment of first chartered colonies.
Enlightenment ideas about liberty and consent in government.
Colonial experience with local control of religious institutions (e.g., Congregationalist church).
Salutary neglect/limited enforcement of the Navigation Acts (1651, 1660, and 1663).
Colonists’ beliefs that they had rights to English liberty and representation.
The Glorious Revolution (1688) demonstrated an unpopular monarch could be deposed.
Debt from French and Indian War (Seven Years’ War) led to increased economic regulations.
The Constitution embodied ideas about self-rule and liberty.
Examples of acceptable contextualization:
“In the period before the French and Indian War, the colonies enjoyed a great amount of economic,
political, and social freedom and even had a government as laid out in the Mayflower Compact. Before
the war, there were Navigation Acts that required the colonies only trade with Britain, but they were
not strictly enforced. Because of these freedoms, it was never heard of that the colonies were seeking
independence or self-government.” (The response earned 1 point for contextualization because it
describes broader historical patterns prior to the time period that established a predisposition to self-
government among the British North American colonies.)
“The ideas of self-rule originally came from the Enlightenment movement. Notable figures such as
John Locke and Thomas Hobbes came up with ideas of liberty and the people ruling their own
government.” (The response earned 1 point for contextualization because it describes the intellectual
movements that created a foundation for ideas about self-government in British North American colonies.)
Example of unacceptable contextualization:
In 1754 the British colonies have expanded along the east coast and have begun their own political
system of government but were still under control from the monarchy of Great Britain.” (The response
did not earn the point for contextualization because it is too broad.)
AP
®
UNITED STATES HISTORY
2019 SCORING GUIDELINES
© 2019 The College Board.
Visit the College Board on the web: collegeboard.org.
Question 2Long Essay Question (continued)
C. Evidence (02 points)
Evidence
Responses earn 1 point by providing at least two specific examples of evidence relevant to the topic of the
prompt. Responses can earn this point without earning the point for a thesis statement.
These examples of evidence must be different from the information used to earn the point for contextualization.
Typically, statements credited as contextualization will be more general statements that place an argument, or
a significant portion of it, in a broader context. Statements credited as evidence will typically be more specific
information.
Examples of evidence used might include:
Virginia House of Burgesses
Mayflower Compact
New England town meetings
Fundamental Orders of Connecticut
Colonial charters
English liberties
British constitution
Albany Congress (1754)
Seven Years’ War (French and Indian War), 17541763
Pontiac’s Rebellion (1763)
Royal Proclamation of 1763
Virtual representation
Sons of Liberty (Massachusetts)
First and Second Continental Congress
State constitution making
Committees of safety
Committees of correspondence
Nonimportation/nonconsumption
Quartering Act (1765)
Vice admiralty courts
The Declaration of Rights and Grievances (1765)
Stamp Act and protests (17651766)
Stamp Act Congress
“No taxation without representation”
Declaratory Act (1766) authority to pass any law regulating colonies “in all cases whatsoever”
Townshend Act (1767)
Boston Massacre (1770)
Tea Act (1773)British East India Co. can sell to colonies without tax
Boston Tea Party (1773)
Coercive/Intolerable Acts (1773)
First and Second Continental Congress
Olive Branch Petition (1775)
Thomas Paine, Common Sense (1776)
Declaration of Independence (1776)
AP
®
UNITED STATES HISTORY
2019 SCORING GUIDELINES
© 2019 The College Board.
Visit the College Board on the web: collegeboard.org.
Question 2 Long Essay Question (continued)
Example of acceptably providing evidence relevant to the topic of the prompt:
The British parliament began to tax goods without the knowledge/consent of the colonists. Political
figures began to write documents and pamphlets such as Thomas Paine’s ‘Common Sense.’” (The
response earned 1 point for evidence. This example correctly identifies the idea of “no taxation without
representationand Thomas Paine’s pamphlet. It did not earn the second evidence point because the
response does not clearly connect the evidence to an argument about self-government.)
Examples of unacceptably providing evidence relevant to the topic of the prompt:
During the 1754’s to 1776’s, America had been left on its own to self govern themselves and was
given an opportunity to establish its own government.” (This example did not earn points for evidence as
it confuses the prompt’s time period with salutary neglect.)
“The Crown was putting a massive tax on the smallest things.” (This example did not earn points for
evidence because it is too broad and vague. More specificity about the kinds of goods taxed and/or the
name of the tax act is necessary to earn this point.)
OR
Supports an Argument
Responses earn 2 points if they support an argument in response to the prompt using specific and relevant
examples of evidence.
Example of acceptable use of evidence to support an argument:
The Stamp Act was the first direct tax on the colonists that they really noticed. The colonists
responded to this with the proclamation of ‘No taxation without Representation,’ the argument that
they could not be taxed if they weren’t represented in Parliament.” (The response earned 2 points for
evidence. It correctly identifies examples in the form of the Stamp Act, which is then used to develop the
larger argument about the relationship between ideas of self-government as a response to British imperial
authority.)
Example of unacceptable use of evidence to support an argument:
The Boston Tea Party is one significant and influential example about the backlash against the British
tea act. Crates of British tea was dumped by the Sons of Liberty, who were activists against tax
collectors.” (The response earned 1 point for providing evidence relevant to the topic of the prompt. This
response did not earn a second evidence point because the Boston Tea Party is not directly connected back
to an argument in response to the prompt.)
AP
®
UNITED STATES HISTORY
2019 SCORING GUIDELINES
© 2019 The College Board.
Visit the College Board on the web: collegeboard.org.
Question 2 Long Essay Question (continued)
D. Analysis and Reasoning (02 points)
Historical Reasoning
Responses earn 1 point by using historical reasoning to frame or structure an argument that addresses the
prompt. To earn this point, the response must demonstrate the use of historical reasoning to frame or structure
an argument, although the reasoning might be uneven or imbalanced.
Examples of using historical reasoning might include:
Explaining how particular ideas from the Enlightenment helped cause American resistance to British
rule, such as the Stamp Act protests or the Boston Tea Party
Tracing the continuity of earlier institutions of self-government such as the Virginia House of
Burgesses to the role of local governing bodies in colonial protests, such as committees of
correspondence
Examples of acceptable use of historical reasoning:
The Boston Tea Party was a protest where colonists dumped British tea into Boston harbor to go
against the burdensome tea tax that Parliament imposed. In response, Britain seized the port, shut
down the Massachusetts legislature, and replaced the former, elected officials with unelected
bureaucrats from England.(The response earned 1 point for historical reasoning because it demonstrates
causation. It did not earn an additional point for complexity as the response did not corroborate, qualify, or
modify an argument.)
“These ideas of self-government created a stirring in them that first questioned, then resented, then
sought to change their reality.” (The response earned 1 point for historical reasoning for demonstrating
change over time. This theme carries throughout the essay, culminating with this statement. The response
did not earn an additional point for complexity as it did not corroborate, qualify, or modify an argument.)
Examples of unacceptable use of historical reasoning:
“Thomas Paine’s ‘Common Sense’ talks about the idea of being free from a ruler and living in a
balanced society. This gave way to the American revolution with the goal of having a government for
the people, by the people.” (This example did not earn a point for historical reasoning. The response
attempts to demonstrate causation, that Thomas Paine’s writings “gave way” to the American Revolution.
This connection is not fully developed and is not directly related to ideas of self-government, which is the
focus of the prompt.)
“Americans might have been the British if we hadn’t wanted self government so badly. Luckily the
Brits made it clear by demanding their money they chose to spend, inflicted a series of radical tax
prices on our largest imports. At that point it would sound nice to be on your own because Britain was
just taking it too far and temptation to self-government prevailed.” (This example did not earn a point for
historical reasoning. The response attempts to demonstrate causation or change over time, but it lacks
specifics to structure an argument that addresses the prompt.)
AP
®
UNITED STATES HISTORY
2019 SCORING GUIDELINES
© 2019 The College Board.
Visit the College Board on the web: collegeboard.org.
Question 2 Long Essay Question (continued)
OR
Complexity
Responses earn 2 points for demonstrating a complex understanding of the topic, using evidence to
corroborate, qualify, or modify that argument.
Demonstrating complex understanding might include:
Explaining a nuance of an issue by analyzing multiple variables by, for example, assessing how ideas
of self-government played out in different colonies such as Virginia and Massachusetts
Explaining multiple causes of American resistance to British colonial authority, such as the
development of a unique American identity
Explaining relevant and insightful connections within and across periods by, for example, making
comparisons with arguments over federalism in the 1780s and 1790s or states’ rights in the 1850s or
1960s
Confirming the validity of an argument by corroborating multiple perspectives across themes by, for
example, exploring cultural ideas about British American exceptionalism or the socioeconomic
differences of British American societies from Great Britain
Qualifying or modifying an argument by considering diverse or alternative views or evidence by, for
example, examining the reasons why colonial resistance against the Navigation Acts and British
imperial supervision before the 1760s was limited compared to the colonial reaction against the
exertion of British imperial authority after 1763
This understanding must be part of the argument, not merely a phrase or reference.
Examples of acceptable demonstration of a complex complex understanding:
The following response earned the point for complexity. While the essay acknowledges the central role self-
government played in leading to revolution, this example identifies that not all colonists (like the Quakers)
adhered to these ideas. This qualifies the response’s overall argument by highlighting specific demographic
groups that did not necessarily support self-government. The response demonstrates a complex
understanding of the topic. “However, despite the mass colonial patriotism, some colonists were
unaffected by ideas of self-governance. Most notable among these is anti-war Pennsylvania
Representative John Dickinson, a Quaker. He, rather than voting to declare independence and go to
war, pushes through the Olive Branch Petition, which was an attempt to reconcile with Britain.”
The following response earned the point for complexity. The response recognizes that debates regarding
self-governance continued well after the end of the war by including references to continued arguments over
federalism (Federalists and Anti-Federalists). This corroborates the original argument by explaining
relevant and insightful connections within and across time periods. The response demonstrates a complex
understanding that the question of self-governance was not settled by the American Revolution. “After
winning independence from Britain at the battle of Yorktown and the signing of the Treaty of Paris,
debates with the intent of protecting self-governance still ensued. Examples such as the debates
between the Federalists and Anti-Federalists over the self-governance of states embodied the lasting
desires that were born on this continent so many centuries ago.”
AP
®
UNITED STATES HISTORY
2019 SCORING GUIDELINES
© 2019 The College Board.
Visit the College Board on the web: collegeboard.org.
Question 2 Long Essay Question (continued)
The following response earned the point for complexity. The essay argues that while not all colonists
initially viewed British imperial tendencies as immoral on their own, an increasing desire for expanded
democracy and self-governance fueled the American Revolution. It identifies the reasons why resistance to
the Navigation Acts was limited in comparison to later taxes such as the Stamp Act. This corroborates the
original argument by showing similarities and differences over time. The response demonstrates a complex
understanding of the topic. “Prior to the American Revolution, taxes from Britain were not foreign to the
colonists. Import duties paid to the Crown and restrictions on trade through the Navigation Acts had
been commonplace. Import duties, however, had been an external tax; they were not paid directly by
individuals, but rather by businesses at the port. The Stamp Act of 1754 did not follow this precedent
of external taxes from Britain. As an internal tax, the Act demanded extra payment on not just stamps,
but papers and other items as well.”
The following response earned the point for complexity. The essay argues that the American Revolution did
not occur because of a great desire for self-government. Rather, the response contends, revolution was
inspired by mercantilism. The response provides evidence throughout of colonial rebellions in response to
excessive taxation and economic hardships. This modifies the original prompt by acknowledging the minor
role of self-government in fomenting rebellion. The response demonstrates a complex understanding of the
topic.
o “Concepts of self-government had some influence on American reactions to the British. However,
the philosophy of mercantilism was the major influence on the American’s Declaration of
Independence. Ideas of self-government had little effect on American resistance to the British.”
o “Conversely, the British use of mercantilism against the colonists was the major influence for the
American colonial reaction. Rebellions occurred in response to the British economic policies such
as the Boston Tea Party. By dumping an entire shipment of tea into Boston Harbor, the residents of
Boston defied the British because of their disturbance of the American economy, not Americas’
ideas of self-government.”
Examples of unacceptable demonstration of a complex understanding:
The following example did not earn the complexity point. The response attempts to corroborate the
argument by demonstrating how debates regarding self-government continued after the American
Revolution. It provides evidence by referencing the sectional crisis in the 1850s; however, a brief reference
is not sufficient to demonstrate a complex understanding of the topic. “The southern states tried breaking
away in the same fashion in the time between 1850 and 1860, leading to the Civil War.”
The following example did not earn the complexity point. The response attempts to modify the topic by
analyzing multiple issues, but it entirely dismisses the validity of the prompt itself and ignores earlier
historical developments of self-government in the British North American colonies. The response makes no
attempt to identify examples of ideas of self-government and therefore does not demonstrate a complex
understanding of the time period. “The reactions of the colonists to the various acts of British imperial
authority from 17561776 was never rooted out of the ideals for self-government. The idea of self-
government only came at the offset of the war. The reactions of the colonists came more from wanting
to be in the period of freedom that salutary neglect brought.”
AP
®
UNITED STATES HISTORY
2019 SCORING GUIDELINES
© 2019 The College Board.
Visit the College Board on the web: collegeboard.org.
Question 3 — Long Essay Question
Evaluate the extent to which debates over slavery in the period from 1830 to 1860 led the United States into the
Civil War.
Maximum Possible Points: 6
Points
Rubric
Notes
A: Thesis/Claim
(01)
Thesis/Claim: Responds to the prompt
with a historically defensible thesis/claim
that establishes a line of reasoning.
(1 point)
To earn this point, the thesis must make a
claim that responds to the prompt rather
than restating or rephrasing the prompt. The
thesis must consist of one or more sentences
located in one place, either in the
introduction or the conclusion.
The thesis must make a historically defensible claim
that establishes a line of reasoning about how
debates over slavery in the period from 1830 to 1860
led the United States into the Civil War.
Examples that earn this point include:
“Overall the debate of slavery had a
tremendous impact on causing the Civil War
as sectionalism and debates over state’s
rights arised.”
“Although tangential debates over issues
such as state vs. federal power may have
contributed to the divide between the North
and the South and thus the outlook of the
Civil War, the primary cause of the Civil War
was the tension over slavery.”
“Throughout the mid-1800’s, debates rose on
the institution of slavery which eventually
led to the Civil War: social arguments were
made such as whites were superior to
blacks; the south argued that slaves were
economically beneficial due to the stable
labor force; and most importantly, political
divisions between the North and the South
greatly caused the Civil War.”
AP
®
UNITED STATES HISTORY
2019 SCORING GUIDELINES
© 2019 The College Board.
Visit the College Board on the web: collegeboard.org.
Question 3 Long Essay Question (continued)
B: Contextualization
(01)
Contextualization: Describes a broader
historical context relevant to the prompt.
(1 point)
To earn this point, the response must relate
the topic of the prompt to broader historical
events, developments, or processes that
occur before, during, or continue after the
time frame of the question. This point is not
awarded for merely a phrase or a reference.
To earn the point, the response must accurately
describe a context relevant to the ways in which
debates over slavery in the period from 1830 to 1860
led the United States into the Civil War.
Examples of context might include the following, with
appropriate elaboration:
Origins of slavery in the British colonial
economy
Virginia and Kentucky Resolutions/Compact
Theory
The United States Constitution’s treatment
of slavery
Ban on the international slave trade in 1808
Britain banned the slave trade in 1807 and
abolished slavery in its colonies in 1833.
Implications of the Missouri Compromise
Southern economic reliance on slavery
Cotton gin
Southern concerns about protectionist trade
policies, rebellions by enslaved people, and
propaganda
The Civil War (18611865)
Debates over states’ rights after the Civil
War
Sharecropping in the South
Reconstruction
Black Codes/Jim Crow
Populism
World War I and World War II
AP
®
UNITED STATES HISTORY
2019 SCORING GUIDELINES
© 2019 The College Board.
Visit the College Board on the web: collegeboard.org.
Question 3 Long Essay Question (continued)
C: Evidence (02)
Evidence: Provides specific examples of
evidence relevant to the topic of the
prompt. (1 point)
To earn the first point, the response must
identify specific historical examples of
evidence relevant to the topic of the prompt.
OR
Supports an Argument: Supports an
argument in response to the prompt using
specific and relevant examples of
evidence. (2 points)
To earn the second point, the response must
use specific historical evidence to support an
argument in response to the prompt.
Examples of evidence used might include:
Abolitionist movement
Nullification Crisis (18321833)
Nat Turner
Harriet Tubman
Sojourner Truth
Compromise of 1850
Stephen Douglas
Frederick Douglass
Underground Railroad
Dred Scott v. Sandford (1857)
Free Soil Party
Gag rule
William Lloyd Garrison
Cotton economy/Southern agriculture
Northern industrialization/urbanization
Wilmot Proviso (1846)
Kansas–Nebraska Act (1854)
Fugitive Slave Act (1850)
Uncle Tom’s Cabin (1852)
Abraham Lincoln
MexicanAmerican War (18461848)
Republican Party
John Brown
Bleeding Kansas
James K. Polk
Texas
Crittenden Compromise
Brooks/Sumner caning (1856)
Manifest Destiny
Sectionalism
Secession (South Carolina)
Second Great Awakening
Ostend Manifesto (1854)
Popular Sovereignty
Andrew Jackson
John C. Calhoun
Filibusters
AP
®
UNITED STATES HISTORY
2019 SCORING GUIDELINES
© 2019 The College Board.
Visit the College Board on the web: collegeboard.org.
Question 3 Long Essay Question (continued)
D: Analysis and Reasoning (02)
Historical Reasoning: Uses historical
reasoning (e.g., comparison, causation,
continuity and change over time) to frame
or structure an argument that addresses
the prompt. (1 point)
To earn the first point, the response must
demonstrate the use of historical reasoning
to frame or structure an argument, although
the reasoning might be uneven or
imbalanced.
OR
Complexity: Demonstrates a complex
understanding of the historical
development that is the focus of the
prompt, using evidence to corroborate,
qualify, or modify an argument that
addresses the question. (2 points)
To earn the second point, the response must
demonstrate a complex understanding. This
can be accomplished in a variety of ways,
such as:
Explaining a nuance of an issue by
analyzing multiple variables
Explaining both similarity and
difference, or explaining both continuity
and change, or explaining multiple
causes, or explaining both causes and
effects
Explaining relevant and insightful
connections within and across periods
Confirming the validity of an argument
by corroborating multiple perspectives
across themes
Qualifying or modifying an argument by
considering diverse or alternative views
or evidence
This understanding must be part of the
argument, not merely a phrase or reference.
Examples of using historical reasoning to frame or
structure an argument might include:
Explaining how the activism of abolitionists
and policies of the Republican Party helped
cause increased tensions between the North
and the South
Tracing the long-term continuity/change in
debates over the spread of the institution of
slavery to the West from the 1830s to 1860
Developing the similarities/differences
between Northern industrialization and
Southern plantation agriculture
OR
Ways of demonstrating a complex understanding of
this prompt might include the following:
Explaining a nuance of an issue by analyzing
multiple variables by, for example, assessing
different arguments against slavery and
different proposals for what to do about it
Explaining both short- and long-term causes
by, for example, addressing the immediate
effects of antislavery activism in the 1830s
and long-term effects by the late 1850s
Explaining relevant and insightful
connections within and across periods by,
for example, comparing proslavery
arguments from the 1830s to 1860 with
arguments about Jim Crow from the 1880s to
the 1900s
Confirming the validity of an argument by
corroborating multiple perspectives across
themes, for example, by considering
Northerners’ and Southerners’ cultural
notions about their unique regional identities
Qualifying or modifying an argument by
considering diverse or alternative views or
evidence by, for example, arguing
convincingly with evidence that debates over
slavery were secondary to debates about
state sovereignty in the outbreak of the Civil
War
If response is completely blank, enter - - for all four score categories: A, B, C, and D.
AP
®
UNITED STATES HISTORY
2019 SCORING GUIDELINES
© 2019 The College Board.
Visit the College Board on the web: collegeboard.org.
Question 3 Long Essay Question (continued)
Scoring Notes
Introductory notes:
Except where otherwise noted, each point of these rubrics is earned independently, e.g., a student
could earn a point for evidence without earning a point for thesis/claim.
Accuracy: The components of these rubrics require that students demonstrate historically defensible
content knowledge. Given the timed nature of the exam, essays may contain errors that do not detract
from their overall quality, as long as the historical content used to advance the argument is accurate.
Clarity: Exam essays should be considered first drafts and thus may contain grammatical errors.
Those errors will not be counted against a student unless they obscure the successful demonstration of
the content knowledge, skills, and practices described below.
Note: Student samples are quoted verbatim and may contain grammatical errors.
A. Thesis/Claim (01 point)
Responses earn 1 point by responding to the prompt with a historically defensible claim that establishes a line
of reasoning about the topic. To earn this point, the thesis must make a claim that responds to the prompt
rather than simply restating or rephrasing the prompt. The thesis must suggest at least one main line of
argument development or establish the analytic categories of the argument.
The thesis must consist of one or more sentences located in one place, either in the introduction or the
conclusion.
Examples of acceptable theses:
“In general, the hostilities were worsened through events like the Compromise of 1850, the Kansas-
Nebraska Act, and Bleeding Kansas until a breaking point with the election of President Lincoln that
led to southern secession and war.” (The response makes a claim in response to the prompt and suggests
an evaluative line of argument development.)
“The issue of slavery therefore did help lead to the Civil War, which is evident with the Compromise of
1850 and the Kansas-Nebraska issue, but it was not the sole reason for the war’s outbreak, for
economic and sectional differences were a big factor as well.(The response makes a claim in response
to the prompt and establishes the analytic categories for the argument.)
Example of unacceptable theses:
“Throughout our history, the US has grown and grown, expanded and expanded. Our culture tends to
move with us as we go, but when the movement and expansion of slavery came into question,
numerous issues arose.(This response attempts to establish a defensible line of reasoning, but it is
vague.)
“Throughout the time period of 1830 until 1860 debate over slavery was the main issue in American
civilization.” (This response largely restates the prompt.)
AP
®
UNITED STATES HISTORY
2019 SCORING GUIDELINES
© 2019 The College Board.
Visit the College Board on the web: collegeboard.org.
Question 3 Long Essay Question (continued)
B. Contextualization (01 point)
Responses earn 1 point by describing a broader historical context relevant to the topic of the prompt. To earn
this point, the response must accurately and explicitly connect the context of the prompt to broader historical
events, developments, or processes that occurred before, during, or continued after the time frame of the
question. This point is not awarded for merely a phrase or reference.
To earn the point, the response must accurately describe a context relevant to how debates over slavery in the
period from 1830 to 1860 led the United States into the Civil War.
Examples might include the following, with appropriate elaboration:
Origins of slavery in the British colonial economy
Virginia and Kentucky Resolutions/Compact theory
The United States Constitution’s treatment of slavery
Ban on the international slave trade in 1808
Britain banned the slave trade in 1807 and abolished slavery in its colonies in 1833
Implications of the Missouri Compromise
Southern economic reliance on slavery
Cotton gin
Southern concerns about protectionist trade policies, rebellions by enslaved people, and propaganda
The Civil War (18611865)
Debates over states’ rights after the Civil War
Sharecropping in the South
Reconstruction
Black Codes/Jim Crow
Populism
World War I and World War II
Example of acceptable contextualization:
“….the political and economic status of slaves during the debates over slavery didn’t change the reality
of lives for blacks after the Civil War. Southerners still argued for control over the Black population
through the Black Codes and Jim Crow laws.” (The response earned 1 point for contextualization
because it establishes a historical pattern that relates to the main idea of the prompt.)
Example of unacceptable contextualization:
Throughout the nineteenth century the economies of the North and the South became increasingly
different.” (While the response has the potential to provide underlying context to the prompt, it does not
provide a clear explanation of what caused the economic differences nor how those changes relate to
debates over slavery.)
C. Evidence (02 points)
Evidence
Responses earn 1 point by providing at least two specific examples of evidence relevant to the topic of the
prompt. Responses can earn this point without earning the point for a thesis statement.
AP
®
UNITED STATES HISTORY
2019 SCORING GUIDELINES
© 2019 The College Board.
Visit the College Board on the web: collegeboard.org.
Question 3 Long Essay Question (continued)
These examples of evidence must be different from the information used to earn the point for contextualization.
Typically, statements credited as contextualization will be more general statements that place an argument, or
a significant portion of it, in a broader context. Statements credited as evidence will typically be more specific
information.
Examples of evidence used might include:
Abolitionist Movement
Nullification Crisis (18321833)
Nat Turner
Harriet Tubman
Sojourner Truth
Compromise of 1850
Stephen Douglas
Frederick Douglass
Underground Railroad
Dred Scott v. Sandford (1857)
Free Soil Party
Gag rule
William Lloyd Garrison
Cotton economy/Southern agriculture
Northern industrialization/urbanization
Wilmot Proviso (1846)
Kansas–Nebraska Act (1854)
Fugitive Slave Act (1850)
Uncle Tom’s Cabin (1852)
Abraham Lincoln
MexicanAmerican War (18461848)
Republican Party
John Brown
Bleeding Kansas
James K. Polk
Texas
Crittenden Compromise
Brooks/Sumner Caning (1856)
Manifest Destiny
Sectionalism
Secession (South Carolina)
Second Great Awakening
Ostend Manifesto (1854)
Popular Sovereignty
Andrew Jackson
John C. Calhoun
Filibusters
AP
®
UNITED STATES HISTORY
2019 SCORING GUIDELINES
© 2019 The College Board.
Visit the College Board on the web: collegeboard.org.
Question 3 Long Essay Question (continued)
Example of acceptably providing evidence relevant to the topic of the prompt:
The passage of the Kansas-Nebraska Act overturned previous precedent prohibiting the expansion of
slavery north of 36’30”, and allowed the inhabitants of a territory to decide whether or not to permit
slavery.(The response earned 1 point by citing evidence relevant to the topic, but it does not use that
evidence to support an argument in response to the prompt, so it did not earn the second point.)
Example of unacceptably providing evidence relevant to the topic of the prompt:
“During the period of 18301860, the United States government passed legislation such as the 3/5
Compromise, the Alien and Sedition Act, and the Missouri Compromise to calm the slavery issue.”
(The response did not earn the point because the evidence presented is not directly relevant to the issues of
slavery and the beginning of the Civil War, nor is it from the period.)
OR
Supports an Argument
Responses earn 2 points if they support an argument in response to the prompt using specific and relevant
examples of evidence.
Example of acceptable use of evidence to support an argument:
“Radical abolitionists such as John Brown attempted to challenge slavery by arming enslaved people
against slaveholders. Although Brown was unsuccessful, Southern leaders used calls by abolitionists
to end slavery as a justification for threatening secession if antislavery laws were passed.” (The
response earned 1 point for historical evidence and 1 point for supporting an argument because it
establishes a specific historical example as contributing to the outbreak of the Civil War.)
Example of unacceptable use of evidence to support an argument:
“Cotton production rapidly increased throughout the nineteenth century as a result of increased
demand for textiles, and new technologies made growing cotton more profitable in regions with
slaves.” (The response cites evidence relevant to the topic, which would count for the first evidence point,
but it does not use that evidence to support an argument about the relationship between debates over
slavery and the origin of the Civil War.)
D. Analysis and Reasoning (02 points)
Historical Reasoning
Responses earn 1 point by using historical reasoning to frame or structure an argument that addresses the
prompt. To earn this point, the response must demonstrate the use of historical reasoning to frame or structure
an argument, although the reasoning might be uneven or imbalanced.
Examples of using historical reasoning might include:
Explaining how the activism of abolitionists and policies of the Republican Party helped cause
increased tensions between the North and the South
Tracing the long-term continuity/change in debates over the spread of the institution of slavery to the
West from the 1830s to 1860
Developing the similarities/differences between Northern industrialization and Southern plantation
agriculture
AP
®
UNITED STATES HISTORY
2019 SCORING GUIDELINES
© 2019 The College Board.
Visit the College Board on the web: collegeboard.org.
Question 3 Long Essay Question (continued)
Example of acceptable use of historical reasoning:
“Although the movement was largely unsuccessful and was focused on issues of trade and taxation,
the Nullification Crisis set the stage for later proslavery arguments. Those in favor of nullification
argued that states had the right to override federal laws, and Southern states in particular argued that
they could disregard any laws placing limitations on the rights of slaveholders.” (The response earned 1
point because it connects the continuation of broader ideas behind nullification to the specific arguments
regarding slavery that contributed to Southern secession and the beginning of the Civil War.)
Example of unacceptable use of historical reasoning:
“Uncle Tom’s cabin was so gruesom, graphic, and realistic that it shocked many northerners. Many
people who read the book began to fight for the abolitionist movement.” (The response did not earn the
point because it did not frame or structure any argument that addresses the topic of the prompt.)
OR
Complexity
Responses earn 2 points for demonstrating a complex understanding of the topic, using evidence to
corroborate, qualify, or modify that argument.
Demonstrating a complex understanding might include:
Explaining a nuance of an issue by analyzing multiple variables by, for example, assessing different
arguments against slavery and different proposals for how to address it
Explaining both short- and long-term causes by, for example, addressing the immediate effects of
antislavery activism in the 1830s and long-term effects seen by the late 1850s
Explaining relevant and insightful connections within and across periods by, for example, comparing
proslavery arguments from the 1830s to 1860 with arguments about Jim Crow from the 1880s to the
1900s
Confirming the validity of an argument by corroborating multiple perspectives across themes, for
example, by considering Northerners’ and Southerners’ cultural notions about their unique regional
identities
Qualifying or modifying an argument by considering diverse or alternative views or evidence by, for
example, arguing convincingly with evidence that debates over slavery were secondary to debates
about state sovereignty in the outbreak of the Civil War
This understanding must be part of the argument, not merely a phrase or reference.
Examples of acceptable demonstration of a complex understanding:
The following response uses nuance to qualify the main thrust of the prompt, asserting that connections
between the North and West also contributed to the Civil War. It makes an overall argument focused on
westward expansion, but it shows complexity in arguing that northern industrialization was heavily reliant
on that expansion and that an industrial North tied to the West threatened the South beyond the question of
slavery. To support this argument, it uses evidence related to the more political aspects of westward
expansion, such as the KansasNebraska Act and the LincolnDouglas Debates, and evidence related to
the more economic ties between the North and West, such as the Erie Canal and raw materials from the
AP
®
UNITED STATES HISTORY
2019 SCORING GUIDELINES
© 2019 The College Board.
Visit the College Board on the web: collegeboard.org.
Question 3 Long Essay Question (continued)
West feeding the industrial factories of the North. “While slave disputes were a major cause that led the
U.S. into the Civil War to some extent, it was also the fact that the North and West were becoming
increasingly connected during the expansion westward, which set conditions for industrialization
which the South saw as a threat to their economy.”
The following response modifies the prompt to address both slavery and the Civil War as the effects of
existing sectionalism, rather than debates over slavery as the cause of the Civil War. It addresses the
Mexican-American War, the Compromise of 1850, the KansasNebraska Act, and the LincolnDouglas
debates to support a nuanced argument. This consideration of alternative views demonstrates a complex
understanding of historical developments from 1830 to 1860.In essence, rather than the debates over
slavery being the main cause of the Civil War, the main cause was that the North and South were
already divided and slavery is just the result.”
The following essay argues that the differences in ideals and sectional interests, and the inability to
compromise during this period, culminated in an irreversible divide. It identifies the reasons for
sectionalism and for the change in the Northern goals in the Civil War, from preserving the Union to ending
slavery. This corroborates the initial argument by demonstrating a causation relation, proving the thesis.
“While issues such as tariffs that supported Northern manufacturing & hurt Southern economies
played a role in creating sectional divide, it was slavery that truly led the United States into Civil War &
the failing of several compromises; many problems that divided the North & South were revolved
around slavery such as the Fugitive Slave Act & Dred Scott Decision, & with the election of republican
Abraham Lincoln, the issues culminated into the secession of the South.” The essay continues, “The
debate would eventually drive the union’s motive for the war, as Lincoln’s Gettysburg address would
shift the focus of the war from unification to emancipating African Americans from the moral evils of
slavery. This would be crucial, as the South would lose support from Britain & Europe as they did not
want to support an immoral cause. Overall, the difference in ideals, sectional interests & inability to
compromise during this period culminated in an irreversible divide between North & South and our
nation’s only Civil War.
Example of unacceptable demonstration of a complex understanding:
The following response does not demonstrate a complex understanding. It attempts unsuccessfully to link
several events across time periods on a theme of inequality. Ultimately, the chronological confusion and
lack of relevant elaboration fail to make the connection. “In the case Plessy vs Ferguson, it was upheld
that segregation was constitutional, which only added to heinous government decisions like the Three
Fifths compromise, which only counted every 3 out of 5 slaves as human beings. This inequality
angered many people, as they argued that every man is equal, as demonstrated in the Declaration of
Independence. The neglect of egalitarianism seemed unconstitutional, but no one with government
power bothered to make any changes.”
AP
®
UNITED STATES HISTORY
2019 SCORING GUIDELINES
© 2019 The College Board.
Visit the College Board on the web: collegeboard.org.
Question 4 — Long Essay Question
Evaluate the extent to which ideas about democracy contributed to the African American Civil Rights
movement in the period from 1940 to 1970.
Maximum Possible Points: 6
Points
Rubric
Notes
A: Thesis/Claim
(01)
Thesis/Claim: Responds to the prompt
with a historically defensible thesis/claim
that establishes a line of reasoning.
(1 point)
To earn this point, the thesis must make a
claim that responds to the prompt rather
than restating or rephrasing the prompt. The
thesis must consist of one or more sentences
located in one place, either in the
introduction or the conclusion.
The thesis must make a historically defensible claim
that establishes a line of reasoning about how ideas
about democracy contributed to the African
American Civil Rights movement from 1940 to 1970.
Examples that earn this point include:
“The ideas about democracy such as the
notion that everyone should be treated
equally and the rights to assemble and
petition greatly contributed to the African
American Civil Rights movement from 1940-
1970.”
“From 1940 to 1970, many ideas of
democracy and equality were all a part to
give way to equity during the Civil Rights
movement, through the ideas that separate is
not equal, voting rights, and tackling racism
through non-violence & Black Power.”
B: Contextualization
(01)
Contextualization: Describes a broader
historical context relevant to the prompt.
(1 point)
To earn this point, the response must relate
the topic of the prompt to broader historical
events, developments, or processes that
occur before, during, or continue after the
time frame of the question. This point is not
awarded for merely a phrase or a reference.
To earn the point, the response must accurately
describe a context relevant to how ideas about
democracy contributed to the African American Civil
Rights movement from 1940 to 1970.
Examples of context might include the following, with
appropriate elaboration:
Earlier African American activism for greater
rights and against vigilante violence
between the 1910s and 1930s (beginning of
what some scholars called a “long Civil
Rights movement”)
African American movement to urban areas
in the North and West during the Great
Migration
African American participation in
World War II
Decolonization in Africa and Asia after
World War II
Cold War competition with the Soviet Union
to win allies and to present itself as a model
society
AP
®
UNITED STATES HISTORY
2019 SCORING GUIDELINES
© 2019 The College Board.
Visit the College Board on the web: collegeboard.org.
Question 4 Long Essay Question (continued)
C: Evidence (02)
Evidence: Provides specific examples of
evidence relevant to the topic of the
prompt. (1 point)
To earn the first point, the response must
identify specific historical examples of
evidence relevant to the topic of the prompt.
OR
Supports an Argument: Supports an
argument in response to the prompt using
specific and relevant examples of
evidence. (2 points)
To earn the second point, the response must
use specific historical evidence to support an
argument in response to the prompt.
Examples of evidence used might include:
Groups:
Congress of Racial Equality (CORE)
National Organization for the Advancement
of Colored People (NAACP)
Southern Christian Leadership Conference
(SCLC)
Student Non-Violent Coordinating
Committee (SNCC)
Black Panthers
Nation of Islam
Dixiecrats
Tuskegee Airmen
People:
John L. Lewis
Rosa Parks
Stokely Carmichael
Martin Luther King, Jr.
Malcolm X
Ruby Bridges
Emmitt Till
George Wallace
Marian Anderson
Events and Ideas:
Brown v. Board of Education of Topeka (1954)
Civil Rights Act of 1964
Voting Rights Act of 1965
Freedom Rides
Montgomery (AL) bus boycott
Little Rock (AR) Central High School
integration
Letter from Birmingham Jail
Greensboro Sit-ins
Southern Manifesto
Birmingham Children’s Crusade
Selma/Bloody Sunday
Black Power
Jim Crow/segregation
Double V Campaign
Civil disobedience
Redlining
Military desegregation
AP
®
UNITED STATES HISTORY
2019 SCORING GUIDELINES
© 2019 The College Board.
Visit the College Board on the web: collegeboard.org.
Question 4 Long Essay Question (continued)
D: Analysis and Reasoning (02)
Historical Reasoning: Uses historical
reasoning (e.g., comparison, causation,
continuity and change over time) to frame
or structure an argument that addresses
the prompt. (1 point)
To earn the first point, the response must
demonstrate the use of historical reasoning
to frame or structure an argument, although
the reasoning might be uneven or
imbalanced.
OR
Complexity: Demonstrates a complex
understanding of the historical
development that is the focus of the
prompt, using evidence to corroborate,
qualify, or modify an argument that
addresses the question. (2 points)
To earn the second point, the response must
demonstrate a complex understanding. This
can be accomplished in a variety of ways,
such as:
Explaining a nuance of an issue by
analyzing multiple variables
Explaining both similarity and
difference, or explaining both continuity
and change, or explaining multiple
causes, or explaining both causes and
effects
Explaining relevant and insightful
connections within and across periods
Confirming the validity of an argument
by corroborating multiple perspectives
across themes
Qualifying or modifying an argument by
considering diverse or alternative views
or evidence
This understanding must be part of the
argument, not merely a phrase or reference.
Examples of using historical reasoning to frame or
structure an argument might include:
Explaining how protests for voting rights
such as the Selma march helped cause
concrete changes in the law and voting
practices
Explaining continuity by tracing the long-
term strategies pursued by Civil Rights
activists to oppose and end segregation
OR
Ways of demonstrating a complex understanding of
this prompt might include the following:
Explaining a nuance of an issue by analyzing
multiple variables by, for example, assessing
how ideas of democracy in the Civil Rights
movement differed in the North and in the
South
Explaining both short- and long-term causes
by, for example, addressing the immediate
causes of the Brown decision and analyzing
how the outcome of the decision evolved
over subsequent decades
Explaining relevant and insightful
connections within and across periods by,
for example, making comparisons with the
Reconstruction era
Confirming the validity of an argument by
corroborating multiple perspectives across
themes by, for example, considering how
foreign relations during the Cold War
affected how the United States government
responded to the Civil Rights movement
Qualifying or modifying an argument by
considering diverse or alternative views or
evidence by, for example, considering how
ideas about Black nationalism and anti-
imperialism influenced leaders of the Civil
Rights movement
If response is completely blank, enter - - for all four score categories: A, B, C, and D.
AP
®
UNITED STATES HISTORY
2019 SCORING GUIDELINES
© 2019 The College Board.
Visit the College Board on the web: collegeboard.org.
Question 4 Long Essay Question (continued)
Scoring Notes
Introductory notes:
Except where otherwise noted, each point of these rubrics is earned independently, e.g., a student
could earn a point for evidence without earning a point for thesis/claim.
Accuracy: The components of these rubrics require that students demonstrate historically defensible
content knowledge. Given the timed nature of the exam, essays may contain errors that do not detract
from their overall quality, as long as the historical content used to advance the argument is accurate.
Clarity: Exam essays should be considered first drafts and thus may contain grammatical errors.
Those errors will not be counted against a student unless they obscure the successful demonstration of
the content knowledge, skills, and practices described below.
Note: Student samples are quoted verbatim and may contain grammatical errors.
A. Thesis/Claim (01 point)
Responses earn 1 point by responding to the prompt with a historically defensible claim that establishes a line
of reasoning about the topic. To earn this point, the thesis must make a claim that responds to the prompt
rather than simply restating or rephrasing the prompt. The thesis must suggest at least one main line of
argument development or establish the analytic categories of the argument.
The thesis must consist of one or more sentences located in one place, either in the introduction or the
conclusion.
Examples of acceptable theses:
“The ideas about democracy such as the notion that everyone should be treated equally and the rights
to assemble and petition greatly contributed to the African American Civil Rights movement from
19401970.(The response makes an evaluative and historically defensible claim that addresses the
prompt and establishes a line of reasoning.)
“From 1940 to 1970, many ideas of democracy and equality were all a part to give way to equity during
the Civil Rights movement, through the ideas that separate is not equal, voting rights, and tackling
racism through non-violence & Black Power.” (The response makes a historically defensible claim that
addresses the prompt, identifies categories of analysis, and establishes a line of reasoning.)
Example of unacceptable theses:
“Ideas on democracy had a large impact on the African American Civil Rights movement.” (This
example largely restates the prompt. While it provides an evaluative adjective, the response does not
establish a historically defensible claim.)
“During the latter part of the 1900s democracy and what side was correct led to or contributed to
African American Civil Rights movement in the sense that it allowed for or encouraged individuals to
fight for what he or she desires.” (The response is too vague to make a historically defensible claim.)
AP
®
UNITED STATES HISTORY
2019 SCORING GUIDELINES
© 2019 The College Board.
Visit the College Board on the web: collegeboard.org.
Question 4 Long Essay Question (continued)
B. Contextualization (01 point)
Responses earn 1 point by describing a broader historical context relevant to the topic of the prompt. To earn
this point, the response must accurately and explicitly connect the context of the prompt to broader historical
events, developments, or processes that occurred before, during, or continued after the time frame of the
question. This point is not awarded for merely a phrase or reference.
To earn the point, the response must accurately describe a context relevant to how ideas about democracy
contributed to the African American Civil Rights movement from 1940 to 1970.
Examples might include the following, with appropriate elaboration:
Earlier African American activism for greater rights and against vigilante violence between the 1910s
and 1930s (beginning of what some scholars called a “long Civil Rights movement”)
African American movement to urban areas in the North and West during the Great Migration
African American participation in World War II
Decolonization in Africa and Asia after World War II
Cold War competition with the Soviet Union to win allies and to present the United States as a model
society
Example of acceptable contextualization:
“The 13
th
Amendment abolished slavery, the 14
th
Amendment made everyone born in the US a citizen
and granted all citizens equal protection under the law, and the 15
th
Amendment gave all male citizens
the right to vote. However, things like the Plessy v. Ferguson ruling, which ruled ‘separate but equal’
constitutional, Jim Crow laws, and literacy tests and poll taxes limited democratic rights of African
Americans for almost 100 years after the Civil War. … Black Americans’ democratic right of equal
protection under the law, constituted by the 14
th
Amendment was being infringed upon, and thus
democracy was not being carried out to its fullest potential.” (The response earned 1 point by describing
the broader historical context of Reconstruction-era Supreme Court decisions and Jim Crow segregation,
which are relevant to the topic of the prompt.)
Example of unacceptable contextualization:
“Through the span of a few centuries they would slowly gain, lose, regain and fight for freedom,
liberty, and independence that was supposed to be promised to them.(The response did not earn a
point because while it indicates an extremely general historical context for the African American Civil
Rights movement, that African Americans experienced gains and losses in terms of freedom over “a few
centuries,” it does not provide any detail about the context, nor does it relate the context to ideas of
democracy described in the prompt.)
C. Evidence (02 points)
Evidence
Responses earn 1 point by providing at least two specific examples of evidence relevant to the topic of the
prompt. Responses can earn this point without earning the point for a thesis statement.
These examples of evidence must be different from the information used to earn the point for contextualization.
Typically, statements credited as contextualization will be more general statements that place an argument, or
a significant portion of it, in a broader context. Statements credited as evidence will typically be more specific
information.
AP
®
UNITED STATES HISTORY
2019 SCORING GUIDELINES
© 2019 The College Board.
Visit the College Board on the web: collegeboard.org.
Question 4 Long Essay Question (continued)
Examples of evidence used might include:
Groups:
Congress of Racial Equality (CORE)
National Organization for the Advancement of Colored People (NAACP)
Southern Christian Leadership Committee (SCLC)
Student Non-Violent Coordinating Conference (SNCC)
Black Panthers
Nation of Islam
Dixiecrats
Tuskegee Airmen
People:
John L. Lewis
Rosa Parks
Stokely Carmichael
Martin Luther King, Jr.
Malcolm X
Ruby Bridges
Emmitt Till
George Wallace
Marian Anderson
Events and Ideas:
Brown v. Board of Education of Topeka (1954)
Civil Rights Act of 1964
Voting Rights Act of 1965
Freedom Rides
Montgomery (AL) bus boycott
Little Rock (AR) Central High School integration
Letter from Birmingham Jail
Greensboro Sit-ins
Southern Manifesto
Birmingham Children’s Crusade
Selma/Bloody Sunday
Black Power
Jim Crow/segregation
Double V Campaign
Civil Disobedience
Redlining
Military Desegregation
Example of acceptably providing evidence relevant to the topic of the prompt:
“One event was the March on Washington. It was led by Martin Luther King. The people that attended
were people of all races. That was also the day he gave his speech. Their was also two groups (SCLC,
SNCC) that did nonviolent protest during the Civil Rights movement.” (The example cites evidence
relevant to the topic, so it earned 1 evidence point, but it does not use that evidence to support an argument
in response to the prompt.)
AP
®
UNITED STATES HISTORY
2019 SCORING GUIDELINES
© 2019 The College Board.
Visit the College Board on the web: collegeboard.org.
Question 4 Long Essay Question (continued)
Example of unacceptably providing evidence relevant to the topic of the prompt
“In addition, there were protests against white owners for mistreating the African American slaves.
Martin Luther King was also part of the abolitionists. He was the tongue of all African Americans who
can’t speak because of fear.” (The response did not earn the point for historical evidence because of the
major error that places MLK in the abolitionist movement.)
OR
Supports an Argument
Responses earn 2 points if they support an argument in response to the prompt using specific and relevant
examples of evidence.
Example of acceptable use of evidence to support an argument:
“Activist organizations like the NAACP, Student Non-Violent Coordiance Committee, and the Southern
Christian Leadership Conference sought to challenge civil acceptance of racial terror through non-
violent organizing. Organizations like the Black Panther Party in the late 60s used principles of
communal organization and violent means if necessary in order to imagine a new version of democracy
that challenged even the most progressive approaches to racial justice.(The response earned 2 points
for evidence because it uses various civil rights and Black liberation organizations and connects them to
the larger movement of democratic reforms called for by the Civil Rights movement.)
Example of unacceptable use of evidence to support an argument:
“The Black Panther and National Association for the Advancement of Colored People greatly believed
in uplifting descendants from the African community through means of western and cultural
education.” (This example cites evidence relevant to the topic, so it could receive the first evidence point,
but it does not use that evidence to support an argument in response to the prompt.)
D. Analysis and Reasoning (02 points)
Historical Reasoning
Responses earn 1 point by using historical reasoning to frame or structure an argument that addresses the
prompt. To earn this point, the response must demonstrate the use of historical reasoning to frame or structure
an argument, although the reasoning might be uneven or imbalanced.
Examples of using historical reasoning might include:
Explaining how protests for voting rights such as the Selma march helped cause concrete changes in
the law and voting practices
Explaining continuity by tracing the long-term strategies pursued by civil rights activists to oppose and
end segregation
AP
®
UNITED STATES HISTORY
2019 SCORING GUIDELINES
© 2019 The College Board.
Visit the College Board on the web: collegeboard.org.
Question 4 Long Essay Question (continued)
Example of acceptable use of historical reasoning:
“Another important democratic idea to the Civil Rights movement was the right to free speech, ideas,
and protest. When African American Rosa Parks was arrested for not moving to the back (segregated)
part of the bus, it spurred anger amongst the black community. This quickly led to the Montgomery
Bus Boycott, where the black community spoke out against segregated buses and refused to ride them.
This greatly impacted bus business. Similarly, the Greensboro sit-ins protested segregation at eating
facilities when a group of black citizens sat at a white table in a restaurant. Both of these protests were
inspired by the democratic idea to speak freely against problems in society.” (This example earned 1
point for historical reasoning because it uses causation to frame an argument that addresses the prompt.)
“During the period of time lots of things started to change. The government was desegregated along
with schools, buses, & the military. Another reason the ideas about democracy contributed to the
African American Civil Rights movement to a major extent is because today we see de jure segregation
is gone.” (This example earned 1 point for historical reasoning because it structures an argument using
change over time.)
Example of unacceptable use of historical reasoning:
“All they were trying to do was get their education, so what they did was stand up and protest for their
civil rights to get an education and they achieved it but democracy played a role because of the votes of
the American people.” (The response did not earn the point for historical reasoning because the argument
is too vague.)
OR
Complexity
Responses earn 2 points for demonstrating a complex understanding of the topic, using evidence to
corroborate, qualify, or modify that argument.
Demonstrating a complex understanding might include:
Explaining a nuance of an issue by analyzing multiple variables by, for example, assessing how ideas
of democracy in the Civil Rights movement differed in the North and in the South
Explaining both short- and long-term causes by, for example, addressing the immediate causes of the
Brown decision and analyzing how the outcome of the decision evolved over subsequent decades
Explaining relevant and insightful connections within and across periods by, for example, making
comparisons with the Reconstruction era
Confirming the validity of an argument by corroborating multiple perspectives across themes by, for
example, considering how foreign relations during the Cold War affected how the United States
government responded to the Civil Rights movement
Qualifying or modifying an argument by considering diverse or alternative views or evidence by, for
example, considering how ideas about Black nationalism and anti-imperialism influenced leaders of
the Civil Rights movement
This understanding must be part of the argument, not merely a phrase or reference.
AP
®
UNITED STATES HISTORY
2019 SCORING GUIDELINES
© 2019 The College Board.
Visit the College Board on the web: collegeboard.org.
Question 4 Long Essay Question (continued)
Examples of acceptable demonstration of a complex understanding:
The following response earned the point for complexity. The response argues that the pursuit of economic
equality and prosperity are democratic values, and they are driving forces in the African American Civil
Rights movement. This corroborates the original argument by explaining relevant, insightful connections
within and across themes. The response demonstrates a complex understanding that ideas about
democracy also extend to economic issues within the larger Civil Rights movement.
Finally, the limits on
the economic prosperity attained by blacks was limited. With the rise of Levittowns in the 1950s, many
whites fled areas that were being filled with black populations to these suburbs. De facto segrigation
led to no one selling houses to black individuals. This caused major economic desperitties and
backlash from colored individuals. After 1965, the shift in civil rights was economic based. Many
African Americans still worked low skilled jobs and in agriculture. Higher paying jobs were given to
white individuals and left African Americans to suffer in poverty.”
The following response earned the point for complexity. This response argues that civil rights organizers
often worked to subvert popular notions of democracy in order to achieve what they believed was just. This
modifies the original prompt by highlighting specific individuals, court cases, and organizations that
diversified approaches to civil rights through alternative definitions of democratic principles. This response
demonstrates a complex understanding of the topic. “Despite brutal harassment, murder, surveillance,
and assault, the inter-linkages between this movement and the antiwar movements, created a full-court
press on the ideological assumptions that ‘all men are created equal,’ and had empirical success. The
drafting of the Civil Rights bill in 1964, Brown v. Board of 1964, Loving v. Virginia decision in 1967,
and integration of black political figures into the polity were all direct results of the black civil rights
organizers, planners, organizations, and principles that sought to undercut normative conceptions of
democracy and conjure up new black liberation.”
The following response earned the point for complexity. While the response acknowledges the role that
ideas about democracy play in the African American Civil Rights movement, this response identifies that
not all leaders of the movement (such as Malcolm X) embraced democratic principles as a source of
inspiration. This qualifies the response’s overall argument by highlighting specific global revolutions that
did not necessarily embrace American notions of democracy. This response demonstrates a complex
understanding of the topic. “Malcolm X certainly espoused ideas encouraging further divisions in
American society. As part of the black powermovement, Malcolm X cited a rich African American
heritage to encourage the formation of an independent state. Furthermore, Malcolm X referenced a
drive for democracy in several surrounding nations where an oppressed group rose up against there
colonial overlords, notably the earlier slave revolt in Haiti as well as the independence of previously
colonized Latin American and African nations. He used this emergence of democracy in other nations
to encourage African Americans to form an independent nation-state away from the confines of white
society. Thus Malcolm X’s black power movement, drawing on earlier leaders like Marcus Garvey
stressed a separate nation for African Americans by referencing democratic principles in other nations.
Most African American Civil Rights activists, however, focused on American principles of democracy
to ensure they remained united.”
AP
®
UNITED STATES HISTORY
2019 SCORING GUIDELINES
© 2019 The College Board.
Visit the College Board on the web: collegeboard.org.
Question 4 Long Essay Question (continued)
Example of unacceptable demonstration of a complex understanding:
This example did not earn the complexity point. The response attempts to qualify the argument by
indicating that civil rights activism cannot take place outside of democracy. The response makes no
attempt to identify examples of ideas of democracy and therefore does not demonstrate a complex
understanding of the time period. “In conclusion, the way democracy worked in the late 1900s, affected
what events occured, thus the Civil Rights movement occured. Although wanting to be seen as equals,
African Americans would not have acted if not for the democracy then.”